Docsity
Docsity

Prepare for your exams
Prepare for your exams

Study with the several resources on Docsity


Earn points to download
Earn points to download

Earn points by helping other students or get them with a premium plan


Guidelines and tips
Guidelines and tips

Pharmacology And The Nursing Process 9th Edition Questions With Answers Best Rated A+ 202, Exams of Nursing

Pharmacology And The Nursing Process 9th Edition Questions With Answers Best Rated A+ 2023/2024 Updates

Typology: Exams

2022/2023

Available from 07/28/2023

Topnurse01
Topnurse01 🇺🇸

4.9

(7)

1.4K documents

1 / 318

Toggle sidebar

Related documents


Partial preview of the text

Download Pharmacology And The Nursing Process 9th Edition Questions With Answers Best Rated A+ 202 and more Exams Nursing in PDF only on Docsity! TEST BANK Pharmacology and the Nursing Process 9th Edition Linda Lane Lilley, Shelly Rainforth Collins, Julie S. Snyder Contents Chapter 01: The Nursing Process and Drug Therapy...................................................................................4 Chapter 02: Pharmacologic Principles.........................................................................................................8 Chapter 03: Lifespan Considerations.........................................................................................................14 Chapter 04: Cultural, Legal, and Ethical Considerations............................................................................20 Chapter 05: Medication Errors: Preventing and Responding.....................................................................26 Chapter 06: Patient Education and Drug Therapy.....................................................................................29 Chapter 07: Over-the-Counter Drugs and Herbal and Dietary Supplements.............................................34 Chapter 08: Gene Therapy and Pharmacogenomics..................................................................................38 Chapter 09: Photo Atlas of Drug Administration.......................................................................................41 Chapter 10: Analgesic Drugs......................................................................................................................50 Chapter 11: General and Local Anesthetics...............................................................................................57 Chapter 12: Central Nervous System Depressants and Muscle Relaxants.................................................61 Chapter 13: Central Nervous System Stimulants and Related Drugs.........................................................66 Chapter 14: Antiepileptic Drugs.................................................................................................................70 Chapter 15: Antiparkinson Drugs...............................................................................................................76 Chapter 16: Psychotherapeutic Drugs.......................................................................................................81 Chapter 17: Substance Use Disorder.........................................................................................................88 Chapter 18: Adrenergic Drugs...................................................................................................................93 Chapter 19: Adrenergic-Blocking Drugs.....................................................................................................98 Chapter 20: Cholinergic Drugs.................................................................................................................103 Chapter 21: Cholinergic-Blocking Drugs..................................................................................................108 Chapter 22: Antihypertensive Drugs........................................................................................................113 Chapter 23: Antianginal Drugs.................................................................................................................119 Chapter 24: Heart Failure Drugs..............................................................................................................125 Chapter 25: Antidysrhythmic Drugs.........................................................................................................131 Chapter 26: Coagulation Modifier Drugs.................................................................................................137 Chapter 27: Antilipemic Drugs.................................................................................................................143 Chapter 28: Diuretic Drugs......................................................................................................................148 Chapter 29: Fluids and Electrolytes.........................................................................................................154 Chapter 30: Pituitary Drugs.....................................................................................................................160 Chapter 01: The Nursing Process and Drug Therapy 5 b. Planning c. Implementation d. Evaluation ANS: D Monitoring the patient’s progress, including the patient’s response to the medication, is part of the evaluation phase. Planning, implementation, and nursing diagnosis are not illustrated by this example. DIF: COGNITIVE LEVEL: Understanding (Comprehension) TOP: NURSING PROCESS: Evaluation MSC: NCLEX: Safe and Effective Care Environment: Management of Care 4. The nurse is assigned to a patient who is newly diagnosed with type 1 diabetes mellitus. Which statement best illustrates an outcome criterion for this patient? a. The patient will follow instructions. b. The patient will not experience complications. c. The patient will adhere to the new insulin treatment regimen. d. The patient will demonstrate correct blood glucose testing technique. ANS: D “Demonstrating correct blood glucose testing technique” is a specific and measurable outcome criterion. “Following instructions” and “not experiencing complications” are not specific criteria. “Adhering to new regimen” would be difficult to measure. DIF: COGNITIVE LEVEL: Applying (Application) TOP: NURSING PROCESS: Planning MSC: NCLEX: Safe and Effective Care Environment: Management of Care 5. Which activity best reflects the implementation phase of the nursing process for the patient who is newly diagnosed with hypertension? a. Providing education on keeping a journal of blood pressure readings b. Setting goals and outcome criteria with the patient’s input c. Recording a drug history regarding over-the-counter medications used at home d. Formulating nursing diagnoses regarding deficient knowledge related to the new treatment regimen ANS: A Education is an intervention that occurs during the implementation phase. Setting goals and outcomes reflects the planning phase. Recording a drug history reflects the assessment phase. Formulating nursing diagnoses reflects analysis of data as part of planning. DIF: COGNITIVE LEVEL: Applying (Application) TOP: NURSING PROCESS: Implementation MSC: NCLEX: Safe and Effective Care Environment: Management of Care 6. The medication order reads, “Give ondansetron (Zofran) 4 mg, 30 minutes before beginning chemotherapy to prevent nausea.” The nurse notes that the route is missing from the order. What is the nurse’s best action? Chapter 01: The Nursing Process and Drug Therapy 6 a. Give the medication intravenously because the patient might vomit. b. Give the medication orally because the tablets are available in 4-mg doses. c. Contact the prescriber to clarify the route of the medication ordered. d. Hold the medication until the prescriber returns to make rounds. ANS: C A complete medication order includes the route of administration. If a medication order does not include the route, the nurse must ask the prescriber to clarify it. The intravenous and oral routes are not interchangeable. Holding the medication until the prescriber returns would mean that the patient would not receive a needed medication. DIF: COGNITIVE LEVEL: Applying (Application) TOP: NURSING PROCESS: Implementation MSC: NCLEX: Safe and Effective Care Environment: Management of Care 7. When the nurse considers the timing of a drug dose, which factor is appropriate to consider when deciding when to give a drug? a. The patient’s ability to swallow b. The patient’s height c. The patient’s last meal d. The patient’s allergies ANS: C The nurse must consider specific pharmacokinetic/pharmacodynamic drug properties that may be affected by the timing of the last meal. The patient’s ability to swallow, height, and allergies are not factors to consider regarding the timing of the drug’s administration. DIF: COGNITIVE LEVEL: Understanding (Comprehension) TOP: NURSING PROCESS: Assessment MSC: NCLEX: Safe and Effective Care Environment: Management of Care 8. The nurse is performing an assessment of a newly admitted patient. Which is an example of subjective data? a. Blood pressure 158/96 mm Hg b. Weight 255 pounds c. The patient reports that he uses the herbal product ginkgo. d. The patient’s laboratory work includes a complete blood count and urinalysis. ANS: C Subjective data include information shared through the spoken word by any reliable source, such as the patient. Objective data may be defined as any information gathered through the senses or that which is seen, heard, felt, or smelled. A patient’s blood pressure, weight, and laboratory tests are all examples of objective data. DIF: COGNITIVE LEVEL: Understanding (Comprehension) TOP: NURSING PROCESS: Assessment MSC: NCLEX: Safe and Effective Care Environment: Management of Care MULTIPLE RESPONSE Chapter 01: The Nursing Process and Drug Therapy 7 1. When giving medications, the nurse will follow the rights of medication administration. The rights include the right documentation, the right reason, the right response, and the patient’s right to refuse. Which of these are additional rights? (Select all that apply.) a. Right drug b. Right route c. Right dose d. Right diagnosis e. Right time f. Right patient ANS: A, B, C, E, F Additional rights of medication administration must always include the right drug, right dose, right time, right route, and right patient. The right diagnosis is incorrect. DIF: COGNITIVE LEVEL: Remembering (Knowledge) TOP: NURSING PROCESS: Implementation MSC: NCLEX: Safe and Effective Care Environment: Safety and Infection Control 2. Place the phases of the nursing process in the correct order, with 1 as the first phase and 5 as the last phase. (Select all that apply.) a. Planning b. Evaluation c. Assessment d. Implementation e. Nursing Diagnoses ANS: A, B, C, D, E The nursing process is an ongoing process that begins with assessing and continues with diagnosing, planning, implementing, and evaluating. DIF: COGNITIVE LEVEL: Applying (Application) TOP: NURSING PROCESS: General MSC: NCLEX: Safe and Effective Care Environment: Management of Care Chapter 02: Pharmacologic Principles 10 6. When administering drugs, the nurse remembers that the duration of action of a drug is defined as which of these? a. The time it takes for a drug to elicit a therapeutic response b. The amount of time needed to remove a drug from circulation c. The time it takes for a drug to achieve its maximum therapeutic response d. The time period at which a drug’s concentration is sufficient to cause a therapeutic response ANS: D Duration of action is the time during which drug concentration is sufficient to elicit a therapeutic response. The other options do not define duration of action. A drug’s onset of action is the time it takes for the drug to elicit a therapeutic response. A drug’s peak effect is the time it takes for the drug to reach its maximum therapeutic response. Elimination is the length of time it takes to remove a drug from circulation. DIF: COGNITIVE LEVEL: Understanding (Comprehension) TOP: NURSING PROCESS: General MSC: NCLEX: Physiological Integrity: Pharmacological and Parenteral Therapies 7. When reviewing the mechanism of action of a specific drug, the nurse reads that the drug works by selective enzyme interaction. Which of these processes describes selective enzyme interaction? a. The drug alters cell membrane permeability. b. The drug’s effectiveness within the cell walls of the target tissue is enhanced. c. The drug is attracted to a receptor on the cell wall, preventing an enzyme from binding to that receptor. d. The drug binds to an enzyme molecule and inhibits or enhances the enzyme’s action with the normal target cell. ANS: D With selective enzyme interaction, the drug attracts the enzymes to bind with the drug instead of allowing the enzymes to bind with their normal target cells. As a result, the target cells are protected from the action of the enzymes. This results in a drug effect. The actions described in the other options do not occur with selective enzyme interactions. DIF: COGNITIVE LEVEL: Understanding (Comprehension) TOP: NURSING PROCESS: General MSC: NCLEX: Physiological Integrity: Pharmacological and Parenteral Therapies 8. When administering a new medication to a patient, the nurse reads that it is highly protein bound. Assuming that the patient’s albumin levels are normal, the nurse would expect which result, as compared to a medication that is not highly protein bound? a. Renal excretion will be faster. b. The drug will be metabolized quickly. c. The duration of action of the medication will be shorter. d. The duration of action of the medication will be longer. ANS: D Chapter 02: Pharmacologic Principles 11 Drugs that are bound to plasma proteins are characterized by longer duration of action. Protein binding does not make renal excretion faster, does not speed up drug metabolism, and does not cause the duration of action to be shorter. DIF: COGNITIVE LEVEL: Applying (Application) TOP: NURSING PROCESS: Planning MSC: NCLEX: Physiological Integrity: Pharmacological and Parenteral Therapies 9. The patient is experiencing chest pain and needs to take a sublingual form of nitroglycerin. Where does the nurse instruct the patient to place the tablet? a. Under the tongue b. On top of the tongue c. At the back of the throat d. In the space between the cheek and the gum ANS: A Drugs administered via the sublingual route are placed under the tongue. Drugs administered via the buccal route are placed in the space between the cheek and the gum; oral drugs are swallowed. The other options are incorrect. DIF: COGNITIVE LEVEL: Understanding (Comprehension) TOP: NURSING PROCESS: Implementation MSC: NCLEX: Physiological Integrity: Pharmacological and Parenteral Therapies 10. The nurse is administering medications to the patient who is in renal failure resulting from end- stage renal disease. The nurse is aware that patients with kidney failure would most likely have problems with which pharmacokinetic phase? a. Absorption b. Distribution c. Metabolism d. Excretion ANS: D The kidneys are the organs that are most responsible for drug excretion. Renal function does not affect the absorption and distribution of a drug. Renal function may affect metabolism of drugs to a small extent. DIF: COGNITIVE LEVEL: Applying (Application) TOP: NURSING PROCESS: Assessment MSC: NCLEX: Physiological Integrity: Pharmacological and Parenteral Therapies 11. A patient who has advanced cancer is receiving opioid medications around the clock to keep him comfortable as he nears the end of his life. Which term best describes this type of therapy? a. Palliative therapy b. Maintenance therapy c. Empiric therapy d. Supplemental therapy ANS: A Chapter 02: Pharmacologic Principles 12 The goal of palliative therapy is to make the patient as comfortable as possible. It is typically used in the end stages of illnesses when all attempts at curative therapy have failed. Maintenance therapy is used for the treatment of chronic illnesses such as hypertension. Empiric therapy is based on clinical probabilities and involves drug administration when a certain pathologic condition has an uncertain but high likelihood of occurrence based on the patient’s initial presenting symptoms. Supplemental (or replacement therapy) supplies the body with a substance needed to maintain normal function. DIF: COGNITIVE LEVEL: Understanding (Comprehension) TOP: NURSING PROCESS: Implementation MSC: NCLEX: Physiological Integrity: Pharmacological and Parenteral Therapies 12. The patient is complaining of a headache and asks the nurse which over-the-counter medication form would work the fastest to help reduce the pain. Which medication form will the nurse suggest? a. A capsule b. A tablet c. An enteric-coated tablet d. A powder ANS: D Of the types of oral medications listed, the powder form would be absorbed the fastest, thus having a faster onset. The tablet, the capsule, and, finally, the enteric-coated tablet would be absorbed next, in that order. DIF: COGNITIVE LEVEL: Applying (Application) TOP: NURSING PROCESS: Implementation MSC: NCLEX: Physiological Integrity: Pharmacological and Parenteral Therapies 13. The nurse will be injecting a drug into the fatty tissue of the patient’s abdomen. Which route does this describe? a. Intradermal b. Subcutaneous c. Intramuscular d. Transdermal ANS: B Injections into the fatty subcutaneous tissue under the dermal layer of skin are referred to as subcutaneous injections. Injections under the more superficial skin layers immediately underneath the epidermal layer of skin and into the dermal layer are known as intradermal injections. Injections into the muscle beneath the subcutaneous fatty tissue are referred to as intramuscular injections. Transdermal drugs are applied to the skin via an adhesive patch. DIF: COGNITIVE LEVEL: Remembering (Knowledge) TOP: NURSING PROCESS: Implementation MSC: NCLEX: Physiological Integrity: Pharmacological and Parenteral Therapies MULTIPLE RESPONSE Chapter 03: Lifespan Considerations 15 ANS: D The milligram per kilogram formula, based on body weight, is the most common method of calculating doses for pediatric patients. The other options are available methods but are not the most commonly used. Height-to-weight ratio is not used. DIF: COGNITIVE LEVEL: Remembering (Knowledge) TOP: NURSING PROCESS: Implementation MSC: NCLEX: Health Promotion and Maintenance 4. The nurse is assessing a newly admitted 83-year-old patient and determines that the patient is experiencing polypharmacy. Which statement most accurately illustrates polypharmacy? a. The patient is experiencing multiple illnesses. b. The patient uses one medication for an illness several times per day. c. The patient uses over-the-counter drugs for an illness. d. The patient uses multiple medications simultaneously. ANS: D Polypharmacy usually occurs when a patient has several illnesses and takes medications for each of them, possibly prescribed by different specialists who may be unaware of other treatments the patient is undergoing. The other options are incorrect. Polypharmacy addresses the medications taken, not just the illnesses. Polypharmacy means the patient is taking several different medications, not just one, and can include prescription drugs, over-the-counter medications, and herbal products. DIF: COGNITIVE LEVEL: Understanding (Comprehension) TOP: NURSING PROCESS: Assessment MSC: NCLEX: Physiological Integrity: Pharmacological and Parenteral Therapies 5. The nurse is aware that confusion, forgetfulness, and increased risk for falls are common responses in an elderly patient who is taking which type of drug? a. Laxatives b. Anticoagulants c. Sedatives d. Antidepressants ANS: C Sedatives and hypnotics often cause confusion, daytime sedation, ataxia, lethargy, forgetfulness, and increased risk for falls in the elderly. Laxatives, anticoagulants, and antidepressants may cause adverse effects in the elderly, but not the ones specified in the question. DIF: COGNITIVE LEVEL: Understanding (Comprehension) TOP: NURSING PROCESS: Implementation MSC: NCLEX: Safe and Effective Care Environment: Safety and Infection Control 6. For accurate medication administration to pediatric patients, the nurse must take into account which criteria? a. Organ maturity b. Renal output c. Body temperature Chapter 03: Lifespan Considerations 16 d. Height ANS: A To administer medications to pediatric patients accurately, one must take into account organ maturity, body surface area, age, and weight. The other options are incorrect; renal output and body temperature are not considerations, and height alone is not sufficient. DIF: COGNITIVE LEVEL: Understanding (Comprehension) TOP: NURSING PROCESS: Implementation MSC: NCLEX: Health Promotion and Maintenance 7. The nurse recognizes that it is not uncommon for an elderly patient to experience a reduction in the stomach’s ability to produce hydrochloric acid. This change may result in which effect? a. Delayed gastric emptying b. Increased gastric acidity c. Decreased intestinal absorption of medications d. Altered absorption of weakly acidic drugs ANS: D Reduction in the stomach’s ability to produce hydrochloric acid is an aging-related change that results in a decrease in gastric acidity and may alter the absorption of weakly acidic drugs. The other options are not results of reduced hydrochloric acid production. DIF: COGNITIVE LEVEL: Applying (Application) TOP: NURSING PROCESS: Assessment MSC: NCLEX: Health Promotion and Maintenance 8. The nurse is administering drugs to neonates and will consider which factor may contribute the most to drug toxicity? a. The lungs are immature. b. The kidneys are small. c. The liver is not fully developed. d. Excretion of the drug occurs quickly. ANS: C A neonate’s liver is not fully developed and cannot detoxify many drugs. The other options are incorrect. The lungs and kidneys do not play major roles in drug metabolism. Renal excretion is slow, not fast, because of organ immaturity, but this is not the factor that contributes the most to drug toxicity. DIF: COGNITIVE LEVEL: Understanding (Comprehension) TOP: NURSING PROCESS: Planning MSC: NCLEX: Health Promotion and Maintenance 9. An 83-year-old woman has been given a thiazide diuretic to treat mild heart failure. She and her daughter should be told to watch for which problems? a. Constipation and anorexia b. Fatigue, leg cramps, and dehydration c. Daytime sedation and lethargy d. Edema, nausea, and blurred vision Chapter 03: Lifespan Considerations 17 ANS: B Electrolyte imbalance, leg cramps, fatigue, and dehydration are common complications when thiazide diuretics are given to elderly patients. The other options do not describe complications that occur when these drugs are given to the elderly. DIF: COGNITIVE LEVEL: Understanding (Comprehension) TOP: NURSING PROCESS: Planning MSC: NCLEX: Physiological Integrity: Pharmacological and Parenteral Therapies 10. An elderly patient with a new diagnosis of hypertension will be receiving a new prescription for an antihypertensive drug. The nurse expects which type of dosing to occur with this drug therapy? a. Drug therapy will be based on the patient’s weight. b. Drug therapy will be based on the patient’s age. c. The patient will receive the maximum dose that is expected to reduce the blood pressure. d. The patient will receive the lowest possible dose at first, and then the dose will be increased as needed. ANS: D As a general rule, dosing for elderly patients should follow the admonition, “Start low, and go slow,” which means to start with the lowest possible dose (often less than an average adult dose) and increase the dose slowly, if needed, based on patient response. The other responses are incorrect. DIF: COGNITIVE LEVEL: Understanding (Comprehension) TOP: NURSING PROCESS: Planning MSC: NCLEX: Physiological Integrity: Pharmacological and Parenteral Therapies 11. The nurse is trying to give a liquid medication to a -year-old child and notes that the medication has a strong taste. Which technique is the best way for the nurse to give the medication to this child? a. Give the medication with a spoonful of ice cream. b. Add the medication to the child’s bottle. c. Tell the child you have candy for him. d. Add the medication to a cup of milk. ANS: A Ice cream or another nonessential food disguises the taste of the medication. The other options are incorrect. If the child does not drink the entire contents of the bottle, medication is wasted and the full dose is not administered. Using the word candy with drugs may lead to the child thinking that drugs are actually candy. If the medication is mixed with a cup of milk, the child may not drink the entire cup of milk, and the distasteful drug may cause the child to refuse milk in the future. DIF: COGNITIVE LEVEL: Applying (Application) TOP: NURSING PROCESS: Implementation MSC: NCLEX: Physiological Integrity: Pharmacological and Parenteral Therapies Chapter 04: Cultural, Legal, and Ethical Considerations 20 Chapter 04: Cultural, Legal, and Ethical Considerations MULTIPLE CHOICE 1. During the development of a new drug, which would be included in the study by the researcher to prevent any bias or unrealistic expectations of the new drug’s usefulness? a. A placebo b. FDA approval c. Informed consent d. Safety information ANS: A To prevent bias that may occur as a result of unrealistic expectations of an investigational new drug, a placebo is incorporated into the study. The other options are incorrect. FDA approval, if given, does not occur until after phase III. Informed consent is required in all drug studies. Safety information is not determined until the study is under way. DIF: COGNITIVE LEVEL: Understanding (Comprehension) TOP: NURSING PROCESS: General MSC: NCLEX: Safe and Effective Care Environment: Safety and Infection Control 2. A member of an investigational drug study team is working with healthy volunteers whose participation will help to determine the optimal dosage range and pharmacokinetics of the drug. The team member is participating in what type of study? a. Phase I b. Phase II c. Phase III d. Phase IV ANS: A Phase I studies involve small numbers of healthy volunteers to determine optimal dosage range and the pharmacokinetics of the drug. The other phases progressively involve volunteers who have the disease or ailment that the drug is designed to diagnose or treat. DIF: COGNITIVE LEVEL: Applying (Application) TOP: NURSING PROCESS: General MSC: NCLEX: Physiological Integrity: Pharmacological and Parenteral Therapies 3. During discharge patient teaching, the nurse reviews prescriptions with a patient. Which statement is correct about refills for an analgesic that is classified as Schedule C- III? a. No prescription refills are permitted. b. Refills are allowed only by written prescription. c. The patient may have no more than five refills in a 6-month period. d. Written prescriptions expire in 12 months. ANS: C Chapter 04: Cultural, Legal, and Ethical Considerations 21 Schedule C-III medications may be refilled no more than five times in a 6-month period. The patient should be informed of this regulation. No prescription refills are permitted for Schedule C-II drugs. Requiring refills by written prescription only applies to Schedule C-II drugs. Schedule C-III prescriptions (written or oral) expire in 6 months. DIF: COGNITIVE LEVEL: Remembering (Knowledge) TOP: NURSING PROCESS: Implementation MSC: NCLEX: Physiological Integrity: Pharmacological and Parenteral Therapies 4. A patient has been selected as a potential recipient of an experimental drug for heart failure. The nurse knows that when informed consent has been obtained, it indicates which of these? a. The patient has been informed of the possible benefits of the new therapy. b. The patient will be informed of the details of the study as the research continues. c. The patient will receive the actual drug during the experiment. d. The patient has had the study’s purpose, procedures, and the risks involved explained to him. ANS: D Informed consent involves the careful explanation of the purpose of the study, the procedures to be used, and the risks involved. The other options do not describe informed consent. DIF: COGNITIVE LEVEL: Understanding (Comprehension) TOP: NURSING PROCESS: Implementation MSC: NCLEX: Safe and Effective Care Environment: Management of Care 5. For which cultural group must the health care provider respect the value placed on preserving harmony with nature and the belief that disease is a result of ill spirits? a. Hispanics b. Asian Americans c. Native Americans d. African Americans ANS: C Some Native Americans believe in preserving harmony with nature and that disease is a result of ill spirits. The groups listed in the other options do not typically reflect these practices. DIF: COGNITIVE LEVEL: Remembering (Knowledge) TOP: NURSING PROCESS: Assessment MSC: NCLEX: Psychosocial Integrity 6. The nurse is assessing an elderly Hispanic woman who is being treated for hypertension. During the assessment, what is important for the nurse to remember about cultural aspects? a. The patient should be discouraged from using folk remedies and rituals. b. The nurse will expect the patient to value protective bracelets and “root workers” as healers. c. The nurse will remember that the balance among body, mind, and environment is important for this patient’s health beliefs. d. The nurse’s assessment needs to include gathering information regarding religious practices and beliefs regarding medication, treatment, and healing. Chapter 04: Cultural, Legal, and Ethical Considerations 22 ANS: D All beliefs need to be considered clearly so as to prevent a conflict from arising between the goals of nursing and health care and the dictates of a patient’s cultural background. Assessing religious practices and beliefs is part of a thorough cultural assessment. The other options are incorrect. The nurse should not ignore a patient’s cultural practices. The concept of balance among body, mind, and environment and the valuing of protective bracelets and root workers reflect beliefs or practices that usually do not apply to the Hispanic cultural group. DIF: COGNITIVE LEVEL: Applying (Application) TOP: NURSING PROCESS: Assessment MSC: NCLEX: Psychosocial Integrity 7. When reviewing the various schedules of controlled drugs, the nurse knows that which description correctly describes Schedule II drugs? a. Drugs with high potential for abuse that have accepted medical use b. Drugs with high potential for abuse that do not have accepted medical use c. Medically accepted drugs that may cause moderate physical or psychologic dependence d. Medically accepted drugs with limited potential for causing physical or psychologic dependence ANS: A Schedule II drugs are those with high potential for abuse but that have accepted medical use. Drugs that have high potential for abuse but do not have accepted medical use are Schedule I drugs. Medically accepted drugs that may cause moderate physical or psychologic dependence are Schedule III drugs. Medically accepted drugs with limited potential for causing physical or psychologic dependence are Schedule IV and V drugs. DIF: COGNITIVE LEVEL: Remembering (Knowledge) TOP: NURSING PROCESS: General MSC: NCLEX: Physiological Integrity: Pharmacological and Parenteral Therapies 8. The nurse is reviewing facts about pharmacology for a review course. The term legend drug refers to which item? a. Over-the-counter drugs b. Prescription drugs c. Orphan drugs d. Older drugs ANS: B The term legend drug refers to prescription drugs, which were differentiated from over-the- counter drugs by the 1951 Durham-Humphrey Amendment. Orphan drugs are drugs that are developed for rare diseases. The other options are not examples of legend drugs. DIF: COGNITIVE LEVEL: Understanding (Comprehension) TOP: NURSING PROCESS: General MSC: NCLEX: Physiological Integrity: Pharmacological and Parenteral Therapies Chapter 04: Cultural, Legal, and Ethical Considerations 25 f. The patient does not eat pork products for religious reasons. ANS: A, C, D, F The past use of medicines, use of herbal treatments, languages spoken, and religious practices and beliefs are components of a cultural assessment. The other options reflect components of a general medication assessment or health history. DIF: COGNITIVE LEVEL: Applying (Application) TOP: NURSING PROCESS: Assessment MSC: NCLEX: Psychosocial Integrity Chapter 05: Medication Errors: Preventing and Responding 26 Chapter 05: Medication Errors: Preventing and Responding MULTIPLE CHOICE 1. The nurse is reviewing medication errors. Which situation is an example of a medication error? a. A patient refuses her morning medications. b. A patient receives a double dose of a medication because the nurse did not cut the pill in half. c. A patient develops hives after having started an IV antibiotic 24 hours earlier. d. A patient complains of severe pain still present 60 minutes after a pain medication was given. ANS: B A medication error is defined as a preventable adverse drug event that involves inappropriate medication use by a patient or health care provider. The other options are not preventable. The patient’s refusing to take medications and complaining of pain after a medication is given are patient behaviors, and the development of hives is a possible allergic reaction. DIF: COGNITIVE LEVEL: Applying (Application) TOP: NURSING PROCESS: Implementation MSC: NCLEX: Safe and Effective Care Environment: Management of Care 2. The nurse is reviewing a list of verbal medication orders. Which is the proper notation of the dose of the drug ordered? a. Digoxin .125 mg b. Digoxin .1250 mg c. Digoxin 0.125 mg d. Digoxin 0.1250 mg ANS: C Digoxin 0.125 mg illustrates the correct notation with a leading zero before the decimal point. Omitting the leading zero may cause the order to be misread, resulting in a large drug overdose. Digoxin .125 mg and digoxin .1250 mg do not have the leading zero before the decimal point. Digoxin 0.1250 mg has a trailing zero, which also is incorrect. DIF: COGNITIVE LEVEL: Applying (Application) TOP: NURSING PROCESS: Assessment MSC: NCLEX: Safe and Effective Care Environment: Management of Care 3. When given a scheduled morning medication, the patient states, “I haven’t seen that pill before. Are you sure it’s correct?” The nurse checks the medication administration record and verifies that it is listed. Which is the nurse’s best response? a. “It’s listed here on the medication sheet, so you should take it.” b. “Go ahead and take it, and then I’ll check with your doctor about it.” c. “It wouldn’t be listed here if it were not ordered for you!” d. “Let me check on the order first before you take it.” ANS: D Chapter 05: Medication Errors: Preventing and Responding 27 When giving medications, the nurse should always listen to and honor any concerns or doubts expressed by the patient. If the patient doubts an order, the nurse should check the written order and/or check with the prescriber. The other options illustrate that the nurse is not listening to the patient’s concerns. DIF: COGNITIVE LEVEL: Applying (Application) TOP: NURSING PROCESS: Planning MSC: NCLEX: Safe and Effective Care Environment: Management of Care 4. During a period of time when the computerized medication order system was down, the prescriber wrote admission orders, and the nurse is transcribing them. The nurse is having difficulty transcribing one order because of the prescriber’s handwriting. Which is the best action for the nurse to take at this time? a. Ask a colleague what the order says. b. Contact the prescriber to clarify the order. c. Wait until the prescriber makes rounds again to clarify the order. d. Ask the patient what medications he takes at home. ANS: B If a prescriber writes an order that is illegible, the nurse should contact the prescriber for clarification. Asking a colleague is not useful because the colleague did not write the order. Waiting for the prescriber to return is incorrect because it would delay implementation of the order. Asking the patient about medications is incorrect because this question will not clarify the current order written by the prescriber. DIF: COGNITIVE LEVEL: Applying (Application) TOP: NURSING PROCESS: Implementation MSC: NCLEX: Safe and Effective Care Environment: Management of Care 5. When taking a telephone order for a medication, which action by the nurse is most appropriate? a. Verify the order with the charge nurse. b. Call back the prescriber to review the order. c. Repeat the order to the prescriber before hanging up the telephone. d. Ask the pharmacist to double-check the order. ANS: C For telephone or verbal orders, repeat the order back to the prescriber before hanging up the telephone. The other options are incorrect. DIF: COGNITIVE LEVEL: Applying (Application) TOP: NURSING PROCESS: Implementation MSC: NCLEX: Safe and Effective Care Environment: Management of Care MULTIPLE RESPONSE 1. The nurse can prevent medication errors by following which principles? (Select all that apply.) a. Assess for allergies after giving medications. b. Use two patient identifiers before giving medications. Chapter 06: Patient Education and Drug Therapy 30 ANS: B Asking “What medications do you take?” is an open-ended question that will encourage greater clarification and additional discussion with the patient. The other options are examples of closed- ended questions, which prompt only a “yes” or “no” answer and provide limited information. DIF: COGNITIVE LEVEL: Applying (Application) TOP: NURSING PROCESS: Assessment MSC: NCLEX: Safe and Effective Care Environment: Management of Care 4. The nurse is setting up a teaching session with an 85-year-old patient who will be going home on anticoagulant therapy. Which educational strategy would reflect consideration of the age-related changes that may exist with this patient? a. Show a video about anticoagulation therapy. b. Present all the information in one session just before discharge. c. Give the patient pamphlets about the medications to read at home. d. Develop large-print handouts that reflect the verbal information presented. ANS: D Developing large-print handouts addresses altered perception in two ways. First, by using visual aids to reinforce verbal instructions, one addresses the possibility of decreased ability to hear high-frequency sounds. By developing the handouts in large print, one addresses the possibility of decreased visual acuity. Showing a video does not allow discussion of the information; furthermore, the text and print may be small and difficult to read and understand. Presenting all the information in one session before discharge also does not allow for discussion, and the patient may not be able to hear or see the information sufficiently. Because of the possibility of decreased short-term memory and slowed cognitive function, giving pamphlets to read may not be appropriate. DIF: COGNITIVE LEVEL: Applying (Application) TOP: NURSING PROCESS: Implementation MSC: NCLEX: Health Promotion and Maintenance 5. When the nurse teaches a skill such as self-injection of insulin to the patient, what is the best way to set up the teaching/learning session? a. Provide written pamphlets for instruction. b. Show a video, and allow the patient to practice as needed on his own. c. Verbally explain the procedure, and provide written handouts for reinforcement. d. After demonstrating the procedure, allow the patient to do several return demonstrations. ANS: D Return demonstration allows the nurse to evaluate the patient’s newly learned skills. The techniques in the other options are incorrect because those suggestions do not allow for evaluation of the patient’s technique. DIF: COGNITIVE LEVEL: Applying (Application) TOP: NURSING PROCESS: Implementation MSC: NCLEX: Safe and Effective Care Environment: Management of Care Chapter 06: Patient Education and Drug Therapy 31 6. A patient with a new prescription for a diuretic has just reviewed with the nurse how to include more potassium in her diet. This reflects learning in which domain? a. Cognitive b. Affective c. Physical d. Psychomotor ANS: A The cognitive domain refers to problem-solving abilities and may involve recall and knowledge of facts. The affective domain refers to values and beliefs. The term physical does not refer to one of the learning domains. The psychomotor domain involves behaviors such as learning how to perform a procedure. DIF: COGNITIVE LEVEL: Understanding (Comprehension) TOP: NURSING PROCESS: Implementation MSC: NCLEX: Safe and Effective Care Environment: Management of Care 7. During an admission assessment, the nurse discovers that the patient does not speak English. Which is considered the ideal resource for translation? a. A family member of the patient b. A close family friend of the patient c. A translator who does not know the patient d. Prewritten note cards with both English and the patient’s language ANS: C The nurse should communicate with the patient in the patient’s native language if at all possible. If the nurse is not able to speak the patient’s native language, a translator should be made available so as to prevent communication problems, minimize errors, and help boost the patient’s level of trust and understanding of the nurse. In practice, this translator may be another nurse or health care professional, a nonprofessional member of the health care team, or a layperson, family member, adult friend, or religious leader or associate. However, it is best to avoid family members as translators, if possible, because of issues with bias, misinterpretation, and potential confidentiality issues. DIF: COGNITIVE LEVEL: Applying (Application) TOP: NURSING PROCESS: Implementation MSC: NCLEX: Safe and Effective Care Environment: Management of Care 8. The nurse is teaching a 16-year-old patient who has a new diagnosis of type 1 diabetes about blood glucose monitoring and the importance of regulating glucose intake. When developing a teaching plan for this teenager, which of Erikson’s stages of development should the nurse consider? a. Trust versus mistrust b. Intimacy versus isolation c. Industry versus inferiority d. Identity versus role confusion ANS: D Chapter 06: Patient Education and Drug Therapy 32 According to Erikson, the adolescent (12 to 18 years of age) is in the identity versus role confusion stage of development. Trust versus mistrust reflects the infancy stage; intimacy versus isolation reflects the young adulthood stage; and industry versus inferiority reflects the school- age stage of development. DIF: COGNITIVE LEVEL: Understanding (Comprehension) TOP: NURSING PROCESS: Assessment MSC: NCLEX: Psychosocial Integrity 9. A 60-year-old patient is on several new medications and expresses worry that she will forget to take her pills. Which action by the nurse would be most helpful in this situation? a. Teaching effective coping strategies b. Asking the patient’s prescriber to reduce the number of drugs prescribed c. Assuring the patient that she will not forget once she is accustomed to the routine d. Assisting the patient with obtaining and learning to use a calendar or pill container ANS: D Calendars, pill containers, or diaries may be helpful to patients who may forget to take prescribed drugs as scheduled. The nurse must ensure that the patient knows how to use these reminder tools. Teaching coping strategies is a helpful suggestion but will not help with remembering to take medications. Asking the prescriber to reduce the number of drugs that are prescribed is not an appropriate action by the nurse. Assuring the patient that she will not forget is false reassurance by the nurse and inappropriate when education is needed. DIF: COGNITIVE LEVEL: Applying (Application) TOP: NURSING PROCESS: Implementation MSC: NCLEX: Physiological Integrity: Reduction of Risk Potential MULTIPLE RESPONSE 1. Which are appropriate considerations when the nurse is assessing the learning needs of a patient? (Select all that apply.) a. Cultural background b. Family history c. Level of education d. Readiness to learn e. Health beliefs ANS: A, C, D, E Family history is not a part of what the nurse considers when assessing learning needs. The other options are appropriate to consider when the nurse is assessing learning needs. DIF: COGNITIVE LEVEL: Understanding (Comprehension) TOP: NURSING PROCESS: Assessment MSC: NCLEX: Safe and Effective Care Environment: Management of Care COMPLETION Chapter 07: Over-the-Counter Drugs and Herbal and Dietary Supplements 35 d. Immunosuppressants ANS: C Valerian may cause increased central nervous system depression if used with sedatives. Digitalis, anticoagulants, and immunosuppressants do not have interactions with valerian. DIF: COGNITIVE LEVEL: Understanding (Comprehension) TOP: NURSING PROCESS: Assessment MSC: NCLEX: Physiological Integrity: Reduction of Risk Potential 4. The patient has been taking an over-the-counter (OTC) acid-reducing drug because he has had “stomach problems” for several months. He tells the nurse that the medicine helps as long as he takes it, but once he stops it, the symptoms return. Which statement by the nurse is the best advice for this patient? a. “The over-the-counter drug has helped you, so you should continue to take it.” b. “The over-the-counter dosage may not be strong enough. You should be taking prescription-strength for best effects.” c. “For best results, you need to watch what you eat in addition to taking this drug.” d. “Using this drug may relieve your symptoms, but it does not address the cause. You should be seen by your health care provider.” ANS: D The use of OTC drugs may postpone effective management of chronic disease states and may delay treatment of serious or life-threatening disorders because these drugs may relieve symptoms without necessarily addressing the cause of the disorder. The other options do not address the need to investigate the cause of the symptoms and are incorrect. DIF: COGNITIVE LEVEL: Applying (Application) TOP: NURSING PROCESS: Planning MSC: NCLEX: Safe and Effective Care Environment: Management of Care 5. During an assessment, the patient tells the nurse that he eats large amounts of garlic for its cardiovascular benefits. Which drug or drug class, if taken, would have a potential interaction with the garlic? a. Acetaminophen (Tylenol) b. Insulin c. Antilipemic drugs d. Sedatives ANS: B The use of garlic may interfere with hypoglycemic drugs. The other options are incorrect because acetaminophen, antilipemic drugs, and sedatives do not have interactions with garlic. DIF: COGNITIVE LEVEL: Understanding (Comprehension) TOP: NURSING PROCESS: Planning MSC: NCLEX: Physiological Integrity: Reduction of Risk Potential Chapter 07: Over-the-Counter Drugs and Herbal and Dietary Supplements 36 6. A patient calls the clinic to ask about taking cranberry dietary supplement capsules because a friend recommended them. The nurse will discuss which possible concern when a patient is taking cranberry supplements? a. It may increase the risk for bleeding if the patient is taking anticoagulants. b. It may increase the risk of toxicity of some psychotherapeutic drugs. c. It may reduce elimination of drugs that are excreted by the kidneys. d. Cranberry may increase the intensity and duration of effects of caffeine. ANS: C The use of cranberry decreases the elimination of many drugs that are renally excreted. The other concerns do not occur with cranberry supplements. DIF: COGNITIVE LEVEL: Applying (Application) TOP: NURSING PROCESS: Planning MSC: NCLEX: Physiological Integrity: Reduction of Risk Potential 7. A patient wants to take the herb gingko to help his memory. The nurse reviews his current medication list and would be concerned about potential interactions if he is taking a medication from which class of drugs? a. Digitalis b. Anticoagulants c. Sedatives d. Immunosuppressants ANS: B The use of gingko increases the risk of bleeding with anticoagulants (warfarin, heparin) and antiplatelets (aspirin, clopidogrel). The other concerns do not occur with gingko supplements. DIF: COGNITIVE LEVEL: Applying (Application) TOP: NURSING PROCESS: Planning MSC: NCLEX: Physiological Integrity: Reduction of Risk Potential MULTIPLE RESPONSE 1. The nurse is conducting a class for senior citizens about the use of over-the-counter (OTC) drugs. Which statements are true regarding the use of OTC drugs? (Select all that apply.) a. Use of OTC drugs may delay treatment of serious ailments. b. Drug interactions with OTC medications are rare. c. OTC drugs may relieve symptoms without addressing the cause of the problem. d. OTC drugs are indicated for long-term treatment of conditions. e. Patients may misunderstand product labels and use the drugs improperly. ANS: A, C, E Chapter 07: Over-the-Counter Drugs and Herbal and Dietary Supplements 37 It is true that use of OTC drugs may delay treatment of serious ailments; OTC drugs may relieve symptoms without addressing the cause of the problem, and patients may misunderstand product labels and use the drugs improperly. These statements should be included when teaching patients about their use. In contrast, drug interactions with OTC medications are not rare and may indeed occur with prescription medications and other OTC drugs. Normally, OTC drugs are intended for short-term treatment of minor ailments. DIF: COGNITIVE LEVEL: Applying (Application) TOP: NURSING PROCESS: Implementation MSC: NCLEX: Physiological Integrity: Pharmacological and Parenteral Therapies 2. The nurse is reviewing the criteria for over-the-counter drugs. Which criteria for over- the- counter status in the United States are accurate? (Select all that apply.) a. The drug must be easy to use. b. The drug must have a low therapeutic index. c. The consumer must be able to monitor the drug’s effectiveness. d. The drug must have a low potential for abuse. e. The drug must not have any interactions with other drugs. ANS: A, C, D In the United States, criteria for over-the-counter status include the drug being easy to use, the drug having a low potential for abuse, and the consumer must be able to monitor the drug’s effectiveness for the condition. The drug must have a high therapeutic index (not a low one), and the drug must have limited interactions with other drugs. DIF: COGNITIVE LEVEL: Applying (Application) TOP: NURSING PROCESS: General MSC: NCLEX: Physiological Integrity: Pharmacological and Parenteral Therapies Chapter 08: Gene Therapy and Pharmacogenomics 40 Alzheimer’s disease. ANS: B, D, E The nurse should assess for factors that may indicate a risk for genetic disorders. A few examples of factors that may indicate a risk for genetic disorders are a higher incidence of a particular disease or disorder in the patient’s family than in the general population; diagnosis of a disease in family members at an unusually young age; or diagnosis of a family member with an unusual form of cancer or with more than one type of cancer. The options regarding heart disease at 60 years of age and cerebral vascular accident at 78 years of age are not factors that indicate a higher risk for genetic disorders. DIF: COGNITIVE LEVEL: Applying (Application) TOP: NURSING PROCESS: Assessment MSC: NCLEX: Physiological Integrity: Reduction of Risk Potential Chapter 09: Photo Atlas of Drug Administration 41 Chapter 09: Photo Atlas of Drug Administration MULTIPLE CHOICE 1. Before administering any medication, what is the nurse’s priority action regarding patient safety? a. Verifying orders with another nurse b. Documenting the medications given c. Counting medications in the medication cart drawers d. Checking the patient’s identification using two identifiers ANS: D Verifying the patient’s identity, using two identifiers, before administering any medication is essential for the patient’s safety and reflects checking one of the “Nine Rights” of medication administration. Documentation is done after the medications are given. DIF: COGNITIVE LEVEL: Applying (Application) TOP: NURSING PROCESS: Assessment MSC: NCLEX: Safe and Effective Care Environment: Safety and Infection Control 2. The nurse is giving an intradermal (ID) injection and will choose which syringe for this injection? a. b. c. d. ANS: B The proper size syringe for ID injection is 1-mL tuberculin. The other syringes pictured are incorrect. Insulin syringes (marked in units) are not used for intradermal injections. DIF: COGNITIVE LEVEL: Analyzing (Analysis) TOP: NURSING PROCESS: Planning MSC: NCLEX: Safe and Effective Care Environment: Safety and Infection Control 3. A patient is to receive a penicillin intramuscular (IM) injection in the ventrogluteal site. The nurse will use which angle for the needle insertion? a. 15 degrees Chapter 09: Photo Atlas of Drug Administration 42 b. 45 degrees c. 60 degrees d. 90 degrees ANS: D The proper angle for IM injections is 90 degrees. The other angles are incorrect. DIF: COGNITIVE LEVEL: Remembering (Knowledge) TOP: NURSING PROCESS: Implementation MSC: NCLEX: Physiological Integrity: Pharmacological and Parenteral Therapies 4. When administering medication by IV bolus (push), the nurse will occlude the IV line by which method? a. Not pinching the IV tubing at all b. Pinching the tubing just above the injection port c. Pinching the tubing just below the injection port d. Pinching the tubing just above the drip chamber of the infusion set ANS: B Before a medication is injected by IV push, the IV line is occluded by pinching the tubing just above the injection port. The other locations are incorrect. DIF: COGNITIVE LEVEL: Understanding (Comprehension) TOP: NURSING PROCESS: Implementation MSC: NCLEX: Physiological Integrity: Pharmacological and Parenteral Therapies 5. The nurse has an order to administer an intramuscular (IM) immunization to a 2-month-old child. Which site is considered the best choice for this injection? a. Deltoid b. Dorsogluteal c. Ventrogluteal d. Vastus lateralis ANS: D The vastus lateralis is the preferred site of injection of drugs such as immunizations for infants. The other sites are not appropriate for infants. The ventrogluteal site is the preferred site for adults and children. The deltoid site is used only for the administration of immunizations to toddlers, older children, and adults (not infants) and only for small volumes of medication. The dorsogluteal site is no longer recommended because of the possibility of nerve injury. DIF: COGNITIVE LEVEL: Understanding (Comprehension) TOP: NURSING PROCESS: Implementation MSC: NCLEX: Physiological Integrity: Pharmacological and Parenteral Therapies 6. The nurse needs to administer insulin subcutaneously to an obese patient. Which is the proper technique for this injection? a. Using the Z-track method b. Inserting the needle at a 5- to 15-degree angle until resistance is felt c. Pinching the skin at the injection site, and then inserting the needle to below the Chapter 09: Photo Atlas of Drug Administration 45 a. Discarding the medications and repeating preparation b. Asking the patient if he will take the medications c. Waiting until the next dose time, and then giving the medications d. Retrieving the medications and administering them to avoid waste ANS: A Medications that fall to the floor must be discarded, and the procedure must be repeated with new medications. The other actions are not appropriate. DIF: COGNITIVE LEVEL: Analyzing (Analysis) TOP: NURSING PROCESS: Implementation MSC: NCLEX: Safe and Effective Care Environment: Safety and Infection Control 13. When giving a buccal medication to a patient, which action by the nurse is appropriate? a. Encouraging the patient to swallow, if necessary b. Administering water after the medication has been given c. Placing the medication between the upper or lower molar teeth and the cheek d. Placing the tablet under the patient’s tongue and allowing it to dissolve completely ANS: C Buccal medications are properly administered between the upper or lower molar teeth and the cheek. Caution the patient against swallowing, and do not administer with water. Medications given under the tongue are sublingually administered. DIF: COGNITIVE LEVEL: Understanding (Comprehension) TOP: NURSING PROCESS: Implementation MSC: NCLEX: Physiological Integrity: Pharmacological and Parenteral Therapies 14. The nurse is giving medications through a percutaneous endoscopic gastrostomy (PEG) tube. Which technique is correct? a. Administering the medications using a 3-mL medication syringe b. Applying firm pressure on the syringe’s piston to infuse the medication c. Flushing the tubing with 30 mL of saline after the medication has been given d. Using the barrel of the syringe, allowing the medication to flow via gravity into the tube ANS: D For PEG tubes (and nasogastric tubes), medications are poured into the barrel of the syringe with the piston removed, and the medication is allowed to flow via gravity into the tube. Fluid must never be forced into the tube. The tubing is to be flushed with 30 mL of tap water (not saline) to ensure that the medication is cleared from the tube after the medication has been given. A 3-mL syringe is too small for this procedure. DIF: COGNITIVE LEVEL: Understanding (Comprehension) TOP: NURSING PROCESS: Implementation MSC: NCLEX: Physiological Integrity: Pharmacological and Parenteral Therapies 15. The nurse is about to give a rectal suppository to a patient. Which technique would facilitate the administration and absorption of the rectal suppository? Chapter 09: Photo Atlas of Drug Administration 46 a. Having the patient lie on his or her right side, unless contraindicated b. Having the patient hold his or her breath during insertion of the medication c. Lubricating the suppository with a small amount of petroleum-based lubricant before insertion d. Encouraging the patient to lie on his or her left side for 15 to 20 minutes after insertion ANS: D Position the patient on his or her left side for rectal suppository insertion. The suppository is then lubricated with a small amount of water-soluble lubricant, not petroleum-based substances. The patient is told to take a deep breath and exhale through the mouth during insertion. Then the patient needs to remain lying on the left side for 15 to 20 minutes to allow absorption of the drug. DIF: COGNITIVE LEVEL: Applying (Application) TOP: NURSING PROCESS: Implementation MSC: NCLEX: Physiological Integrity: Pharmacological and Parenteral Therapies 16. A patient is receiving eyedrops that contain a beta-blocker medication. The nurse will use what method to reduce systemic effects after administering the eyedrops? a. Wiping off excess liquid immediately after instilling the drops b. Having the patient close the eye tightly after the drops are instilled c. Having the patient try to keep the eye open for 30 seconds after the drops are instilled d. Applying gentle pressure to the patient’s nasolacrimal duct for 30 to 60 seconds after instilling the drops ANS: D When administering ophthalmic drugs that may cause systemic effects, one’s finger should be protected by a clean tissue or glove and gentle pressure applied to the patient’s nasolacrimal duct for 30 to 60 seconds. The other actions are not appropriate. DIF: COGNITIVE LEVEL: Applying (Application) TOP: NURSING PROCESS: Implementation MSC: NCLEX: Physiological Integrity: Pharmacological and Parenteral Therapies 17. A 2-year-old child is to receive eardrops. The nurse is teaching the parent about giving the eardrops. Which statement reflects the proper technique for administering eardrops to this child? a. Administer the drops without pulling on the ear lobe. b. Straighten the ear canal by pulling the lobe upward and back. c. Straighten the ear canal by pulling the pinna down and back. d. Straighten the ear canal by pulling the pinna upward and outward. ANS: C In an infant or a child younger than 3 years of age, the ear canal is straightened by pulling the pinna down and back. In adults, the pinna is pulled up and outward. Pulling the lobe and administering eardrops without pulling on the ear lobe are not appropriate actions. DIF: COGNITIVE LEVEL: Understanding (Comprehension) TOP: NURSING PROCESS: Implementation Chapter 09: Photo Atlas of Drug Administration 47 MSC: NCLEX: Physiological Integrity: Pharmacological and Parenteral Therapies 18. A patient with asthma is to begin medication therapy using a metered-dose inhaler. What is an important reminder to include during teaching sessions with the patient? a. Repeat subsequent puffs, if ordered, after 5 minutes. b. Inhale slowly while pressing down to release the medication. c. Inhale quickly while pressing down to release the medication. d. Administer the inhaler while holding it 3 to 4 inches away from the mouth. ANS: B Position the inhaler to an open mouth, with the inhaler 1 to 2 inches away from the mouth, or attach a spacer to the mouthpiece of the inhaler, or place the mouthpiece in the mouth. To administer, press down on the inhaler to release the medication while inhaling slowly. Wait 1 to 2 minutes between puffs if a second puff of the same medication has been ordered. DIF: COGNITIVE LEVEL: Applying (Application) TOP: NURSING PROCESS: Implementation MSC: NCLEX: Physiological Integrity: Pharmacological and Parenteral Therapies 19. When giving medications, the nurse will use Standard Precautions, which include what action? a. Bending the needle to prevent reuse b. Recapping needles to prevent needle sticks c. Discarding all syringes and needles in the trash can d. Discarding all syringes and needles in a puncture-resistant container ANS: D Standard Precautions include wearing clean gloves when there is potential exposure to a patient’s blood or other body fluids; never recapping needles; never bending needles or syringes; and discarding all disposable syringes and needles in the appropriate puncture-resistant container. DIF: COGNITIVE LEVEL: Remembering (Knowledge) TOP: NURSING PROCESS: Implementation MSC: NCLEX: Safe and Effective Care Environment: Safety and Infection Control 20. A patient says he prefers to chew rather than swallow his pills. One of the pills has the abbreviation SR behind the name of the medication. The nurse needs to remember which correct instruction regarding how to give this medication? a. Break the tablet into halves or quarters. b. Dissolve the tablet in a small amount of water before giving it. c. Do not crush or break the tablet before administration. d. Crush the tablet as needed to ease administration. ANS: C Sustained-release (SR) and enteric-coated tablets or capsules are forms of medications that must not be crushed before administration so as to protect the gastrointestinal lining or the medication itself. Do not break, dissolve, or crush these tablets before administering. DIF: COGNITIVE LEVEL: Applying (Application) TOP: NURSING PROCESS: Implementation Chapter 10: Analgesic Drugs 50 Chapter 10: Analgesic Drugs MULTIPLE CHOICE 1. A patient was diagnosed with pancreatic cancer last month, and has complained of a dull ache in the abdomen for the past 4 months. This pain has been gradually increasing, and the pain relievers taken at home are no longer effective. What type of pain is the patient experiencing? a. Acute pain b. Chronic pain c. Somatic pain d. Neuropathic pain ANS: B Chronic pain is associated with cancer and is characterized by slow onset, long duration, and dull, persistent aching. The patient’s symptoms are not characteristics of acute pain, somatic pain, or neuropathic pain. DIF: COGNITIVE LEVEL: Understanding (Comprehension) TOP: NURSING PROCESS: Assessment MSC: NCLEX: Physiological Integrity: Basic Care and Comfort 2. An 18-year-old basketball player fell and twisted his ankle during a game. The nurse will expect to administer which type of analgesic? a. Synthetic opioid, such as meperidine (Demerol) b. Opium alkaloid, such as morphine sulfate c. Opioid antagonist, such as naloxone HCL (Narcan) d. Nonopioid analgesic, such as indomethacin (Indocin) ANS: D Somatic pain, which originates from skeletal muscles, ligaments, and joints, usually responds to nonopioid analgesics such as nonsteroidal anti-inflammatory drugs (NSAIDs). The other options are not the best choices for somatic pain. DIF: COGNITIVE LEVEL: Applying (Application) TOP: NURSING PROCESS: Assessment MSC: NCLEX: Physiological Integrity: Basic Care and Comfort 3. A patient is recovering from abdominal surgery, which he had this morning. He is groggy but complaining of severe pain around his incision. What is the most important assessment data to consider before the nurse administers a dose of morphine sulfate to the patient? a. His pulse rate b. His respiratory rate c. The appearance of the incision d. The date of his last bowel movement ANS: B Chapter 10: Analgesic Drugs 51 One of the most serious adverse effects of opioids is respiratory depression. The nurse must assess the patient’s respiratory rate before administering an opioid. The other options are incorrect. DIF: COGNITIVE LEVEL: Applying (Application) TOP: NURSING PROCESS: Assessment MSC: NCLEX: Physiological Integrity: Reduction of Risk Potential 4. A 78-year-old patient is in the recovery room after having a lengthy surgery on his hip. As he is gradually awakening, he requests pain medication. Within 10 minutes after receiving a dose of morphine sulfate, he is very lethargic and his respirations are shallow, with a rate of 7 per minute. The nurse prepares for which priority action at this time? a. Assessment of the patient’s pain level b. Immediate intubation and artificial ventilation c. Administration of naloxone (Narcan) d. Close observation of signs of opioid tolerance ANS: C Naloxone, an opioid-reversal agent, is used to reverse the effects of acute opioid overdose and is the drug of choice for reversal of opioid-induced respiratory depression. This situation is describing an opioid overdose, not opioid tolerance. Intubation and artificial ventilation are not appropriate because the patient is still breathing at 7 breaths/min. It would be inappropriate to assess the patient’s level of pain. DIF: COGNITIVE LEVEL: Applying (Application) TOP: NURSING PROCESS: Implementation MSC: NCLEX: Physiological Integrity: Pharmacological and Parenteral Therapies 5. A patient will be discharged with a 1-week supply of an opioid analgesic for pain management after abdominal surgery. The nurse will include which information in the teaching plan? a. How to prevent dehydration due to diarrhea b. The importance of taking the drug only when the pain becomes severe c. How to prevent constipation d. The importance of taking the drug on an empty stomach ANS: C Gastrointestinal (GI) adverse effects, such as nausea, vomiting, and constipation, are the most common adverse effects associated with opioid analgesics. Physical dependence usually occurs in patients undergoing long-term treatment. Diarrhea is not an effect of opioid analgesics. Taking the dose with food may help minimize GI upset. DIF: COGNITIVE LEVEL: Applying (Application) TOP: NURSING PROCESS: Implementation MSC: NCLEX: Physiological Integrity: Reduction of Risk Potential 6. A patient has been treated for lung cancer for 3 years. Over the past few months, the patient has noticed that the opioid analgesic is not helping as much as it had previously and more medication is needed for the same pain relief. The nurse is aware that this patient is experiencing which of these? Chapter 10: Analgesic Drugs 52 a. Opioid addiction b. Opioid tolerance c. Opioid toxicity d. Opioid abstinence syndrome ANS: B Opioid tolerance is a common physiologic result of long-term opioid use. Patients with opioid tolerance require larger doses of the opioid agent to maintain the same level of analgesia. This situation does not describe toxicity (overdose), addiction, or abstinence syndrome (withdrawal). DIF: COGNITIVE LEVEL: Understanding (Comprehension) TOP: NURSING PROCESS: Evaluation MSC: NCLEX: Physiological Integrity: Pharmacological and Parenteral Therapies 7. A 38-year-old man has come into the urgent care center with severe hip pain after falling from a ladder at work. He says he has taken several pain pills over the past few hours but cannot remember how many he has taken. He hands the nurse an empty bottle of acetaminophen (Tylenol). The nurse is aware that the most serious toxic effect of acute acetaminophen overdose is which condition? a. Tachycardia b. Central nervous system depression c. Hepatic necrosis d. Nephropathy ANS: C Hepatic necrosis is the most serious acute toxic effect of an acute overdose of acetaminophen. The other options are incorrect. DIF: COGNITIVE LEVEL: Understanding (Comprehension) TOP: NURSING PROCESS: Assessment MSC: NCLEX: Safe and Effective Care Environment: Safety and Infection Control 8. A 57-year-old woman being treated for end-stage breast cancer has been using a transdermal opioid analgesic as part of the management of pain. Lately, she has been experiencing breakthrough pain. The nurse expects this type of pain to be managed by which of these interventions? a. Administering NSAIDs b. Administering an immediate-release opioid c. Changing the opioid route to the rectal route d. Making no changes to the current therapy ANS: B If a patient is taking long-acting opioid analgesics, breakthrough pain must be treated with an immediate-release dosage form that is given between scheduled doses of the long-acting opioid. The other options are not appropriate actions. DIF: COGNITIVE LEVEL: Applying (Application) TOP: NURSING PROCESS: Planning MSC: NCLEX: Physiological Integrity: Pharmacological and Parenteral Therapies Chapter 10: Analgesic Drugs 55 15. A patient arrives at the urgent care center complaining of leg pain after a fall when rock climbing. The x-rays show no broken bones, but he has a large bruise on his thigh. The patient says he drives a truck and does not want to take anything strong because he needs to stay awake. Which statement by the nurse is most appropriate? a. “It would be best for you not to take anything if you are planning to drive your truck.” b. “We will discuss with your doctor about taking an opioid because that would work best for your pain.” c. “You can take acetaminophen, also known as Tylenol, for pain, but no more than 1000 mg per day.” d. “You can take acetaminophen, also known as Tylenol, for pain, but no more than 3000 mg per day.” ANS: D Acetaminophen is indicated for mild-to-moderate pain and does not cause drowsiness, as an opioid would. Currently, the maximum daily amount of acetaminophen is 3000 mg/day. The 1000-mg amount per day is too low. Telling the patient not to take any pain medications is incorrect. DIF: COGNITIVE LEVEL: Applying (Application) TOP: NURSING PROCESS: Planning MSC: NCLEX: Physiological Integrity: Pharmacological and Parenteral Therapies 16. A patient is suffering from tendonitis of the knee. The nurse is reviewing the patient’s medication administration record and recognizes that which adjuvant medication is most appropriate for this type of pain? a. Antidepressant b. Anticonvulsant c. Corticosteroid d. Local anesthesia ANS: C Corticosteroids have an anti-inflammatory effect, which may help to reduce pain. The other medications do not have anti-inflammatory properties. DIF: COGNITIVE LEVEL: Understanding (Comprehension) TOP: NURSING PROCESS: Planning MSC: NCLEX: Physiological Integrity: Pharmacological and Parenteral Therapies MULTIPLE RESPONSE 1. Vicodin (acetaminophen/hydrocodone) is prescribed for a patient who has had surgery. The nurse informs the patient that which common adverse effects can occur with this medication? (Select all that apply.) a. Diarrhea b. Constipation c. Lightheadedness d. Nervousness Chapter 10: Analgesic Drugs 56 e.Urinary retention f.Itching ANS: B, C, E, F Constipation (not diarrhea), lightheadedness (not nervousness), urinary retention, and itching are some of the common adverse effects that the patient may experience while taking Vicodin. DIF: COGNITIVE LEVEL: Understanding (Comprehension) TOP: NURSING PROCESS: Implementation MSC: NCLEX: Physiological Integrity: Pharmacological Therapies COMPLETION 1. A patient is to receive codeine, 40 mg subcutaneously, every 6 hours as needed for pain. The solution is available in a concentration of 30 mg/mL. Identify how many milliliters of codeine will be drawn up for this dose. (record answer to one decimal place) ANS: 1.3 mL DIF: COGNITIVE LEVEL: Applying (Application) TOP: NURSING PROCESS: Implementation MSC: NCLEX: Physiological Integrity: Pharmacological and Parenteral Therapies Chapter 11: General and Local Anesthetics 57 Chapter 11: General and Local Anesthetics MULTIPLE CHOICE 1. During a fishing trip, a patient pierced his finger with a large fishhook. He is now in the emergency department to have it removed. The nurse anticipates that which type of anesthesia will be used for this procedure? a. No anesthesia b. Topical benzocaine spray on the area c. Topical prilocaine (EMLA) cream around the site d. Infiltration of the puncture wound with lidocaine ANS: D Infiltration anesthesia is commonly used for minor surgical procedures. It involves injecting the local anesthetic solution intradermally, subcutaneously, or submucosally across the path of nerves supplying the area to be anesthetized. The local anesthetic may be administered in a circular pattern around the operative field. The other types are not appropriate for this injury. This is a painful procedure; therefore, the option of “no anesthesia” is incorrect. DIF: COGNITIVE LEVEL: Applying (Application) TOP: NURSING PROCESS: Planning MSC: NCLEX: Physiological Integrity: Pharmacological and Parenteral Therapies 2. While monitoring a patient who had surgery under general anesthesia 2 hours ago, the nurse notes a sudden elevation in body temperature. This finding may be an indication of which problem? a. Tachyphylaxis b. Postoperative infection c. Malignant hypertension d. Malignant hyperthermia ANS: D A sudden elevation in body temperature during the postoperative period may indicate the occurrence of malignant hyperthermia, a life-threatening emergency. The elevated temperature does not reflect the other problems listed. DIF: COGNITIVE LEVEL: Applying (Application) TOP: NURSING PROCESS: Assessment MSC: NCLEX: Safe and Effective Care Environment: Management of Care 3. When assessing patients in the preoperative area, the nurse knows that which patient is at a higher risk for an altered response to anesthesia? a. The 21-year-old patient who has never had surgery before b. The 35-year-old patient who stopped smoking 8 years ago c. The 40-year-old patient who is to have a kidney stone removed d. The 82-year-old patient who is to have gallbladder removal ANS: D Chapter 11: General and Local Anesthetics 60 COMPLETION 1. A patient is to receive midazolam (Versed) 2 mg IV push over 2 minutes just before an endoscopy procedure. The medication is available in a strength of 1 mg/mL. Identify how many milliliters of medication will the nurse draw up into the syringe for this dose. ANS: 2 mL DIF: COGNITIVE LEVEL: Applying (Application) TOP: NURSING PROCESS: Implementation MSC: NCLEX: Physiological Integrity: Pharmacological and Parenteral Therapies Chapter 12: Central Nervous System Depressants and Muscle Relaxants 61 Chapter 12: Central Nervous System Depressants and Muscle Relaxants MULTIPLE CHOICE 1. A patient who has received some traumatic news is panicking and asks for some medication to help settle down. The nurse anticipates giving which drug that is most appropriate for this situation? a. Diazepam (Valium) b. Zolpidem (Ambien) c. Phenobarbital d. Cyclobenzaprine (Flexeril) ANS: A Benzodiazepines such as diazepam are used as anxiolytics, or sedatives. Zolpidem is used as a hypnotic for sleep. Phenobarbital is not used as an anxiolytic but is used for seizure control. Cyclobenzaprine is a muscle relaxant and is not used to reduce anxiety. DIF: COGNITIVE LEVEL: Applying (Application) TOP: NURSING PROCESS: Planning MSC: NCLEX: Physiological Integrity: Pharmacological and Parenteral Therapies 2. A patient has been taking phenobarbital for 2 weeks as part of his therapy for epilepsy. He tells the nurse that he feels tense and that “the least little thing” bothers him now. Which is the correct explanation for this problem? a. These are adverse effects that usually subside after a few weeks. b. The drug must be stopped immediately because of possible adverse effects. c. This drug causes the rapid eye movement (REM) sleep period to increase, resulting in nightmares and restlessness. d. This drug causes deprivation of REM sleep and may cause the inability to deal with normal stress. ANS: D Barbiturates such as phenobarbital deprive people of REM sleep, which can result in agitation and the inability to deal with normal stress. A rebound phenomenon occurs when the drug is stopped (not during therapy), and the proportion of REM sleep increases, sometimes resulting in nightmares. The other options are incorrect. DIF: COGNITIVE LEVEL: Understanding (Comprehension) TOP: NURSING PROCESS: Evaluation MSC: NCLEX: Physiological Integrity: Basic Care and Comfort 3. A 50-year-old man who has been taking phenobarbital for 1 week is found very lethargic and unable to walk after eating out for dinner. His wife states that he has no other prescriptions and that he did not take an overdose—the correct number of pills is in the bottle. The nurse suspects that which of these may have happened? a. He took a multivitamin. b. He drank a glass of wine. c. He took a dose of aspirin. Chapter 12: Central Nervous System Depressants and Muscle Relaxants 62 d. He developed an allergy to the drug. ANS: B Alcohol has an additive effect when combined with barbiturates and causes central nervous system (CNS) depression. Multivitamins and aspirin do not interact with barbiturates, and this situation does not illustrate an allergic reaction. DIF: COGNITIVE LEVEL: Applying (Application) TOP: NURSING PROCESS: Assessment MSC: NCLEX: Physiological Integrity: Pharmacological and Parenteral Therapies 4. A patient has been taking temazepam (Restoril) for intermittent insomnia. She calls the nurse to say that when she takes it, she sleeps well, but the next day she feels “so tired.” Which explanation by the nurse is correct? a. “Long-term use of this drug results in a sedative effect.” b. “If you take the drug every night, this hangover effect will be reduced.” c. “These drugs affect the sleep cycle, resulting in daytime sleepiness.” d. “These drugs increase the activity of the central nervous system, making you tired the next day.” ANS: C Benzodiazepines suppress REM sleep to a degree (although not as much as barbiturates) and, thus, result in daytime sleepiness (a hangover effect). The other statements are incorrect. DIF: COGNITIVE LEVEL: Applying (Application) TOP: NURSING PROCESS: Implementation MSC: NCLEX: Physiological Integrity: Physiological Adaptation 5. A patient is recovering from a minor automobile accident that occurred 1 week ago. He is taking cyclobenzaprine (Flexeril) for muscular pain and goes to physical therapy three times a week. Which nursing diagnosis would be appropriate for him? a. Risk for injury related to decreased sensorium b. Risk for addiction related to psychologic dependency c. Decreased fluid volume related to potential adverse effects d. Disturbed sleep pattern related to the drug’s interference with REM sleep ANS: A Musculoskeletal relaxants have a depressant effect on the CNS; thus, the patient needs to be taught the importance of taking measures to minimize self-injury and falls related to decreased sensorium. “Risk for addiction” is not a NANDA nursing diagnosis. The other nursing diagnoses are not appropriate for this situation. DIF: COGNITIVE LEVEL: Applying (Application) TOP: NURSING PROCESS: Nursing Diagnosis MSC: NCLEX: Physiological Integrity: Reduction of Risk Potential 6. A patient is taking flurazepam (Dalmane) three to four nights a week for sleeplessness. She is concerned that she cannot get to sleep without taking the medication. What nonpharmacologic measures should the nurse suggest to promote sleep for this patient? Chapter 12: Central Nervous System Depressants and Muscle Relaxants 65 b. Pregnancy c. Epilepsy d. Severe chronic obstructive pulmonary disease e. Severe liver disease f. Diabetes mellitus ANS: B, D, E Contraindications to barbiturates include pregnancy, significant respiratory difficulties, and severe liver disease. The other disorders are not contraindications. DIF: COGNITIVE LEVEL: Applying (Application) TOP: NURSING PROCESS: Assessment MSC: NCLEX: Physiological Integrity: Pharmacological and Parenteral Therapies 2. The barbiturate phenobarbital is prescribed for a patient with epilepsy. While assessing the patient’s current medications, the nurse recognizes that interactions may occur with which drugs? (Select all that apply.) a. Antihistamines b. Opioids c. Diuretics d. Anticoagulants e. Oral contraceptives f. Insulin ANS: A, B, D, E The co-administration of barbiturates and alcohol, antihistamines, benzodiazepines, opioids, and tranquilizers may result in additive CNS depression. Co-administration of anticoagulants and barbiturates can result in decreased anticoagulation response and possible clot formation. Co- administration of barbiturates and oral contraceptives can result in accelerated metabolism of the contraceptive drug and possible unintended pregnancy. There are no interactions with diuretics and insulin. DIF: COGNITIVE LEVEL: Understanding (Comprehension) TOP: NURSING PROCESS: Assessment MSC: NCLEX: Physiological Integrity: Pharmacological Therapies Chapter 13: Central Nervous System Stimulants and Related Drugs 66 Chapter 13: Central Nervous System Stimulants and Related Drugs MULTIPLE CHOICE 1. A patient is receiving instructions regarding the use of caffeine. The nurse shares that caffeine should be used with caution if which of these conditions is present? a. A history of peptic ulcers b. Migraine headaches c. Asthma d. A history of kidney stones ANS: A Caffeine should be used with caution by patients who have histories of peptic ulcers or cardiac dysrhythmias or who have recently had myocardial infarctions. The other conditions are not contraindications to the use of caffeine. DIF: COGNITIVE LEVEL: Understanding (Comprehension) TOP: NURSING PROCESS: Assessment MSC: NCLEX: Physiological Integrity: Pharmacological and Parenteral Therapies 2. A patient who started taking orlistat (Xenical) 1 month ago calls the clinic to report some “embarrassing” adverse effects. She tells the nurse that she has had episodes of “not being able to control my bowel movements.” Which statement is true about this situation? a. These are expected adverse effects that will eventually diminish. b. The patient will need to stop this drug immediately if these adverse effects are occurring. c. The patient will need to increase her fat intake to prevent these adverse effects. d. The patient will need to restrict fat intake to less than 30% to help reduce these adverse effects. ANS: D Restricting dietary intake of fat to less than 30% of total calories can help reduce some of the GI adverse effects, which include oily spotting, flatulence, and fecal incontinence. The other options are incorrect. DIF: COGNITIVE LEVEL: Applying (Application) TOP: NURSING PROCESS: Evaluation MSC: NCLEX: Physiological Integrity: Physiological Adaptation 3. A 6-year-old boy has been started on an extended-release form of methylphenidate hydrochloride (Ritalin) for the treatment of attention deficit hyperactivity disorder (ADHD). During a follow- up visit, his mother tells the nurse that she has been giving the medication at bedtime so that it will be “in his system” when he goes to school the next morning. What is the nurse’s appropriate evaluation of the mother’s actions? a. She is giving him the medication dosage appropriately. b. The medication should not be taken until he is at school. c. The medication should be taken with meals for optimal absorption. d. The medication should be given 4 to 6 hours before bedtime to diminish insomnia. Chapter 13: Central Nervous System Stimulants and Related Drugs 67 ANS: D Central nervous system stimulants should be taken 4 to 6 hours before bedtime to decrease insomnia. Generally speaking, once-a-day dosing is used with extended-release or long-acting preparations. These formulations eliminate the need to take this medication at school. DIF: COGNITIVE LEVEL: Applying (Application) TOP: NURSING PROCESS: Evaluation MSC: NCLEX: Physiological Integrity: Physiological Adaptation 4. A 22-year-old nursing student has been taking NoDoz (caffeine) tablets for the past few weeks to “make it through” the end of the semester and exam week. She is in the university clinic today because she is “exhausted.” What nursing diagnosis may be appropriate for her? a. Noncompliance b. Impaired physical mobility c. Disturbed sleep pattern d. Imbalanced nutrition: less than body requirements ANS: C The main ingredient in NoDoz, caffeine, is a central nervous system stimulant that can be used to increase mental alertness. Restlessness, anxiety, and insomnia are common adverse effects. Thus, disturbed sleep pattern is the most appropriate nursing diagnosis of those listed. DIF: COGNITIVE LEVEL: Applying (Application) TOP: NURSING PROCESS: Nursing Diagnosis MSC: NCLEX: Physiological Integrity: Physiological Adaptation 5. A 10-year-old patient will be started on methylphenidate hydrochloride (Ritalin) therapy. The nurse will perform which essential baseline assessment before this drug is started? a. Eye examination b. Height and weight c. Liver function studies d. Hearing test ANS: B Assessment of baseline height and weight is important before beginning Ritalin therapy because it may cause a temporary slowing of growth in prepubertal children. The other studies are not as essential at this time. DIF: COGNITIVE LEVEL: Understanding (Comprehension) TOP: NURSING PROCESS: Assessment MSC: NCLEX: Physiological Integrity: Pharmacological and Parenteral Therapies 6. When evaluating a patient who is taking orlistat (Xenical), which is an intended therapeutic effect? a. Increased wakefulness b. Increased appetite c. Decreased weight d. Decreased hyperactivity Chapter 14: Antiepileptic Drugs 70 Chapter 14: Antiepileptic Drugs MULTIPLE CHOICE 1. The nurse is reviewing the dosage schedule for several different antiepileptic drugs (AEDs). Which antiepileptic drug allows for once-a-day dosing? a. Levetiracetam (Keppra) b. Phenobarbital c. Valproic acid (Depakote) d. Gabapentin (Neurontin) ANS: B Phenobarbital has the longest half-life of all standard AEDs, including those listed in the other options, so it allows for once-a-day dosing. DIF: COGNITIVE LEVEL: Understanding (Comprehension) TOP: NURSING PROCESS: Assessment MSC: NCLEX: Physiological Integrity: Pharmacological and Parenteral Therapies 2. The nurse has given medication instructions to a patient receiving phenytoin (Dilantin). Which statement by the patient indicates that the patient has an adequate understanding of the instructions? a. “I will need to take extra care of my teeth and gums while on this medication.” b. “I can go out for a beer while on this medication.” c. “I can skip doses if the side effects bother me.” d. “I will be able to stop taking this drug once the seizures stop.” ANS: A Scrupulous dental care is necessary to prevent gingival hypertrophy during therapy with phenytoin. Alcohol and other central nervous system depressants may cause severe sedation. Consistent dosing is important to maintain therapeutic drug levels. Therapy with AEDs usually must continue for life and must not be stopped once seizures stop. DIF: COGNITIVE LEVEL: Analyzing (Analysis) TOP: NURSING PROCESS: Evaluation MSC: NCLEX: Safe and Effective Care Environment: Management of Care 3. When teaching a patient about taking a newly prescribed antiepileptic drug (AED) at home, the nurse will include which instruction? a. “Driving is allowed after 2 weeks of therapy.” b. “If seizures recur, take a double dose of the medication.” c. “Antacids can be taken with the AED to reduce gastrointestinal adverse effects.” d. “Regular, consistent dosing is important for successful treatment.” ANS: D Chapter 14: Antiepileptic Drugs 71 Consistent dosing, taken regularly at the same time of day, at the recommended dose, and with meals to reduce the common gastrointestinal adverse effects, is the key to successful management of seizures when taking AEDs. Noncompliance is the factor most likely to lead to treatment failure. DIF: COGNITIVE LEVEL: Applying (Application) TOP: NURSING PROCESS: Implementation MSC: NCLEX: Safe and Effective Care Environment: Management of Care 4. A patient has a 9-year history of a seizure disorder that has been managed well with oral phenytoin (Dilantin) therapy. He is to be NPO (consume nothing by mouth) for surgery in the morning. What will the nurse do about his morning dose of phenytoin? a. Give the same dose intravenously. b. Give the morning dose with a small sip of water. c. Contact the prescriber for another dosage form of the medication. d. Notify the operating room that the medication has been withheld. ANS: C If there are any questions about the medication order or the medication prescribed, contact the prescriber immediately for clarification and for an order of the appropriate dose form of the medication. Do not change the route without the prescriber’s order. There is an increased risk of seizure activity if one or more doses of the AED are missed. DIF: COGNITIVE LEVEL: Applying (Application) TOP: NURSING PROCESS: Implementation MSC: NCLEX: Physiological Integrity: Pharmacological and Parenteral Therapies 5. The nurse is monitoring a patient who has been taking carbamazepine (Tegretol) for 2 months. Which effects would indicate that autoinduction has started to occur? a. The drug levels for carbamazepine are higher than expected. b. The drug levels for carbamazepine are lower than expected. c. The patient is experiencing fewer seizures. d. The patient is experiencing toxic effects from the drug. ANS: B With carbamazepine, autoinduction occurs and leads to lower than expected drug concentrations. Therefore, the dosage may have to be adjusted with time. The other options are incorrect. DIF: COGNITIVE LEVEL: Understanding (Comprehension) TOP: NURSING PROCESS: Evaluation MSC: NCLEX: Physiological Integrity: Pharmacological and Parenteral Therapies 6. The nurse is giving an intravenous dose of phenytoin (Dilantin). Which action is correct when administering this drug? a. Give the dose as a fast intravenous (IV) bolus. b. Mix the drug with normal saline, and give it as a slow IV push. c. Mix the drug with dextrose (D5W), and give it as a slow IV push. d. Mix the drug with any available solution as long as the administration rate is correct. Chapter 14: Antiepileptic Drugs 72 ANS: B Intravenous phenytoin is given only with normal saline solution to prevent precipitation formation caused by incompatibilities. The IV push dose must be given slowly (not exceeding 50 mg/min in adults), and the patient must be monitored for bradycardia and decreased blood pressure. DIF: COGNITIVE LEVEL: Applying (Application) TOP: NURSING PROCESS: Implementation MSC: NCLEX: Physiological Integrity: Pharmacological and Parenteral Therapies 7. The U.S. Food and Drug Administration has issued a warning for users of antiepileptic drugs. Based on this report, the nurse will monitor for which potential problems with this class of drugs? a. Increased risk of suicidal thoughts and behaviors b. Signs of bone marrow depression c. Indications of drug addiction and dependency d. Increased risk of cardiovascular events, such as strokes ANS: A In December 2008, the U.S. Food and Drug Administration (FDA) required black box warnings on all antiepileptic drugs regarding the risk of suicidal thoughts and behaviors. Patients being treated with antiepileptic drugs for any indication need to be monitored for the emergence or worsening of depression, suicidal thoughts or behavior, or any unusual changes in mood or behavior. The other options are incorrect. DIF: COGNITIVE LEVEL: Understanding (Comprehension) TOP: NURSING PROCESS: Evaluation MSC: NCLEX: Physiological Integrity: Pharmacological and Parenteral Therapies 8. A patient is experiencing status epilepticus. The nurse prepares to give which drug of choice for the treatment of this condition? a. Diazepam (Valium) b. Midazolam (Versed) c. Valproic acid (Depakote) d. Carbamazepine (Tegretol) ANS: A Diazepam (Valium) is considered by many to be the drug of choice for status epilepticus. Other drugs that are used are listed in Table 14-3 and do not include the drugs listed in the other options. DIF: COGNITIVE LEVEL: Understanding (Comprehension) TOP: NURSING PROCESS: Planning MSC: NCLEX: Physiological Integrity: Pharmacological and Parenteral Therapies 9. Phenytoin (Dilantin) has a narrow therapeutic index. The nurse recognizes that this characteristic indicates which of these? a. The safe and the toxic plasma levels of the drug are very close to each other. Chapter 14: Antiepileptic Drugs 75 DIF: COGNITIVE LEVEL: Applying (Application) TOP: NURSING PROCESS: Implementation MSC: NCLEX: Physiological Integrity: Pharmacological and Parenteral Therapies Chapter 15: Antiparkinson Drugs 76 Chapter 15: Antiparkinson Drugs MULTIPLE CHOICE 1. A patient has been taking selegiline (Eldepryl), 20 mg/day for 1 month. Today, during his office visit, he tells the nurse that he forgot and had a beer with dinner last evening, and “felt awful.” What did the patient most likely experience? a. Hypotension b. Hypertension c. Urinary discomfort d. Gastrointestinal upset ANS: B At doses that exceed 10 mg/day, selegiline becomes a nonselective monoamine oxidase inhibitor (MAOI), contributing to the development of the cheese effect, so-called because it interacts with tyramine-containing foods (cheese, red wine, beer, and yogurt) and can cause severe hypertension. DIF: COGNITIVE LEVEL: Understanding (Comprehension) TOP: NURSING PROCESS: Evaluation MSC: NCLEX: Physiological Integrity: Reduction of Risk Potential 2. A patient has been given a prescription for levodopa-carbidopa (Sinemet) for her newly diagnosed Parkinson’s disease. She asks the nurse, “Why are there two drugs in this pill?” The nurse’s best response reflects which fact? a. Carbidopa allows for larger doses of levodopa to be given. b. Carbidopa prevents the breakdown of levodopa in the periphery. c. There are concerns about drug-food interactions with levodopa therapy that do not exist with the combination therapy. d. Carbidopa is the biologic precursor of dopamine and can penetrate into the central nervous system. ANS: B When given in combination with levodopa, carbidopa inhibits the breakdown of levodopa in the periphery and thus allows smaller doses of levodopa to be used. Lesser amounts of levodopa result in fewer unwanted adverse effects. Levodopa, not carbidopa, is the biologic precursor of dopamine and can penetrate into the CNS. DIF: COGNITIVE LEVEL: Applying (Application) TOP: NURSING PROCESS: Implementation MSC: NCLEX: Physiological Integrity: Pharmacological and Parenteral Therapies 3. When a patient is taking an anticholinergic such as benztropine (Cogentin) as part of the treatment for Parkinson’s disease, the nurse should include which information in the teaching plan? a. Minimize the amount of fluid taken while on this drug. b. Discontinue the medication if adverse effects occur. c. Take the medication on an empty stomach to enhance absorption. Chapter 15: Antiparkinson Drugs 77 d. Use artificial saliva, sugarless gum, or hard candy to counteract dry mouth. ANS: D Dry mouth can be managed with artificial saliva through drops or gum, frequent mouth care, forced fluids, and sucking on sugar-free hard candy. Anticholinergics should be taken with or after meals to minimize GI upset and must not be discontinued suddenly. The patient must drink at least 3000 mL/day unless contraindicated. Drinking water is important, even if the patient is not thirsty or in need of hydration, to prevent and manage the adverse effect of constipation. DIF: COGNITIVE LEVEL: Applying (Application) TOP: NURSING PROCESS: Planning MSC: NCLEX: Physiological Integrity: Pharmacological and Parenteral Therapies 4. A patient has been treated with antiparkinson medications for 3 months. What therapeutic responses should the nurse look for when assessing this patient? a. Decreased appetite b. Gradual development of cogwheel rigidity c. Newly developed dyskinesias d. Improved ability to perform activities of daily living ANS: D Therapeutic responses to antiparkinson agents include an improved sense of well-being, improved mental status, increased appetite, increased ability to perform activities of daily living and to concentrate and think clearly, and less intense parkinsonian manifestations. DIF: COGNITIVE LEVEL: Applying (Application) TOP: NURSING PROCESS: Evaluation MSC: NCLEX: Physiological Integrity: Physiological Adaptation 5. Carbidopa-levodopa (Sinemet) is prescribed for a patient with Parkinson’s disease. The nurse informs the patient that which common adverse effects can occur with this medication? a. Drowsiness, headache, weight loss b. Dizziness, insomnia, nausea c. Peripheral edema, fatigue, syncope d. Heart palpitations, hypotension, urinary retention ANS: D Common adverse reactions associated with carbidopa-levodopa include palpitations, hypotension, urinary retention, dyskinesia, and depression. The other effects may occur with other antiparkinson drugs. DIF: COGNITIVE LEVEL: Understanding (Comprehension) TOP: NURSING PROCESS: Implementation MSC: NCLEX: Physiological Integrity: Pharmacological and Parenteral Therapies 6. The nurse is assessing the medication history of a patient with a new diagnosis of Parkinson’s disease. Which condition is a contraindication for the patient, who will be taking tolcapone (Tasmar)? a. Glaucoma Chapter 15: Antiparkinson Drugs 80 e. Concurrent use of monoamine oxidase inhibitors (MAOIs) ANS: A, B, E Angle-closure glaucoma, a history of melanoma or other undiagnosed skin conditions, and concurrent use of MAOIs are contraindications to the use of carbidopa-levodopa. The other options are incorrect. DIF: COGNITIVE LEVEL: Applying (Application) TOP: NURSING PROCESS: Assessment MSC: NCLEX: Physiological Integrity: Reduction of Risk Potential COMPLETION 1. The prescriber writes this order, “Give amantadine (Symmetrel) 100 mg per PEG tube twice a day.” The medication is available in a liquid form with a concentration 50 mg/5 mL. Identify how many milliliters will the nurse give with each dose. ANS: 10 mL DIF: COGNITIVE LEVEL: Applying (Application) TOP: NURSING PROCESS: Implementation MSC: NCLEX: Physiological Integrity: Pharmacological and Parenteral Therapies Chapter 16: Psychotherapeutic Drugs 81 Chapter 16: Psychotherapeutic Drugs MULTIPLE CHOICE 1. The nurse reads in the patient’s medication history that the patient is taking buspirone (BuSpar). The nurse interprets that the patient may have which disorder? a. Anxiety disorder b. Depression c. Schizophrenia d. Bipolar disorder ANS: A Buspirone is indicated for the treatment of anxiety disorders, not depression, schizophrenia, or bipolar disorder. DIF: COGNITIVE LEVEL: Understanding (Comprehension) TOP: NURSING PROCESS: Implementation MSC: NCLEX: Physiological Integrity: Pharmacological and Parenteral Therapies 2. Before beginning a patient’s therapy with selective serotonin reuptake inhibitor (SSRI) antidepressants, the nurse will assess for concurrent use of which medications or medication class? a. Aspirin b. Anticoagulants c. Diuretics d. Nonsteroidal anti-inflammatory drugs ANS: B Use of selective serotonin reuptake inhibitor (SSRI) antidepressants with warfarin results in an increased anticoagulant effect. SSRI antidepressants do not interact with the other drugs or drug classes listed. See Table 16-6 for important drug interactions with SSRIs. DIF: COGNITIVE LEVEL: Understanding (Comprehension) TOP: NURSING PROCESS: Assessment MSC: NCLEX: Physiological Integrity: Reduction of Risk Potential 3. When a patient is receiving a second-generation antipsychotic drug, such as risperidone (Risperdal), the nurse will monitor for which therapeutic effect? a. Fewer panic attacks b. Decreased paranoia and delusions c. Decreased feeling of hopelessness d. Improved tardive dyskinesia ANS: B The therapeutic effects of the antipsychotic drugs include improvement in mood and affect, and alleviation or decrease in psychotic symptoms (decrease in hallucinations, paranoia, delusions, garbled speech). Tardive dyskinesia is a potential adverse effect of these drugs. The other options are incorrect. Chapter 16: Psychotherapeutic Drugs 82 DIF: COGNITIVE LEVEL: Applying (Application) TOP: NURSING PROCESS: Evaluation MSC: NCLEX: Physiological Integrity: Physiological Adaptation 4. A patient has been taking haloperidol (Haldol) for 3 months for a psychotic disorder, and the nurse is concerned about the development of extrapyramidal symptoms. The nurse will monitor the patient closely for which effects? a. Increased paranoia b. Drowsiness and dizziness c. Tremors and muscle twitching d. Dry mouth and constipation ANS: C Extrapyramidal symptoms are manifested by tremors and muscle twitching, and the incidence of such symptoms is high during haloperidol therapy. The other options are incorrect. DIF: COGNITIVE LEVEL: Understanding (Comprehension) TOP: NURSING PROCESS: Assessment MSC: NCLEX: Physiological Integrity: Physiological Adaptation 5. A patient has been taking the monoamine oxidase inhibitor (MAOI) phenelzine (Nardil) for 6 months. The patient wants to go to a party and asks the nurse, “Will just one beer be a problem?” Which advice from the nurse is correct? a. “You can drink beer as long as you have a designated driver.” b. “Now that you’ve had the last dose of that medication, there will be no further dietary restrictions.” c. “If you begin to experience a throbbing headache, rapid pulse, or nausea, you’ll need to stop drinking.” d. “You need to avoid all foods that contain tyramine, including beer, while taking this medication.” ANS: D Foods containing tyramine, such as beer and aged cheeses, should be avoided while a patient is taking an MAOI. Drinking beer while taking an MAOI may precipitate a dangerous hypertensive crisis. The other options are incorrect. DIF: COGNITIVE LEVEL: Analyzing (Analysis) TOP: NURSING PROCESS: Implementation MSC: NCLEX: Safe and Effective Care Environment: Safety and Infection Control 6. A 22-year-old patient has been taking lithium for 1 year, and the most recent lithium level is 0.9 mEq/L. Which statement about the laboratory result is correct? a. The lithium level is therapeutic. b. The lithium level is too low. c. The lithium level is too high. d. Lithium is not usually monitored with blood levels. ANS: A Chapter 16: Psychotherapeutic Drugs 85 b. The patient needs to wear sunscreen while outside because of photosensitivity. c. Long-term therapy may result in nervousness and excitability. d. The medication may be taken with an antacid to reduce gastrointestinal upset. ANS: B Sun exposure and tanning booths need to be avoided with conventional antipsychotics because of the adverse effect of photosensitivity. Instruct the patient to apply sunscreen liberally and to wear sun-protective clothing and hats. DIF: COGNITIVE LEVEL: Applying (Application) TOP: NURSING PROCESS: Implementation MSC: NCLEX: Physiological Integrity: Pharmacological and Parenteral Therapies 13. The nurse is reviewing the food choices of a patient who is taking a monoamine oxidase inhibitor ( MAOI). Which food choice would indicate the need for additional teaching? a. Orange juice b. Fried eggs over-easy c. Salami and Swiss cheese sandwich d. Biscuits and honey ANS: C Aged cheeses, such a Swiss or cheddar cheese, and Salami contain tyramine. Patients who are taking MAOIs need to avoid tyramine-containing foods because of a severe hypertensive reaction that may occur. Orange juice, eggs, biscuits, and honey do not contain tyramine. DIF: COGNITIVE LEVEL: Applying (Application) TOP: NURSING PROCESS: Implementation MSC: NCLEX: Physiological Integrity: Reduction of Risk Potential 14. A patient wants to take a ginseng dietary supplement. The nurse instructs the patient to look for which potential adverse effect? a. Drowsiness b. Palpitations and anxiety c. Dry mouth d. Constipation ANS: B Elevated blood pressure, chest pain or palpitations, anxiety, insomnia, headache, nausea, vomiting, and diarrhea are potential adverse effects of ginseng. Drowsiness, difficulty with urination, and constipation are not potential adverse effects of ginseng. DIF: COGNITIVE LEVEL: Understanding (Comprehension) TOP: NURSING PROCESS: Implementation MSC: NCLEX: Physiological Integrity: Reduction of Risk Potential 15. The nurse is reviewing medications used for depression. Which of these statements is a reason that selective serotonin reuptake inhibitors (SSRIs) are more widely prescribed today than tricyclic antidepressants? a. SSRIs have fewer sexual side effects. Chapter 16: Psychotherapeutic Drugs 86 b. Unlike tricyclic antidepressants, SSRIs do not have drug-food interactions. c. Tricyclic antidepressants cause serious cardiac dysrhythmias if an overdose occurs. d. SSRIs cause a therapeutic response faster than tricyclic antidepressants. ANS: C Death from overdose of tricyclic antidepressants usually results from either seizures or dysrhythmias. SSRIs are associated with significantly fewer and less severe systemic adverse effects, especially anticholinergic and cardiovascular adverse effects. The other options are incorrect. DIF: COGNITIVE LEVEL: Applying (Application) TOP: NURSING PROCESS: Planning MSC: NCLEX: Physiological Integrity: Pharmacological and Parenteral Therapies MULTIPLE RESPONSE 1. A patient who has been taking a selective serotonin reuptake inhibitor (SSRI) is complaining of “feeling so badly” when he started taking an over-the-counter St. John’s wort herbal product at home. The nurse suspects that he is experiencing serotonin syndrome. Which of these are symptoms of serotonin syndrome? (Select all that apply.) a. Agitation b. Drowsiness c. Tremors d. Bradycardia e. Sweating f. Constipation ANS: A, C, E Common symptoms of serotonin syndrome include delirium, agitation, tachycardia, sweating, hyperreflexia, shivering, coarse tremors, and others. See Box 16-1 for a full list of symptoms. DIF: COGNITIVE LEVEL: Understanding (Comprehension) TOP: NURSING PROCESS: Assessment MSC: NCLEX: Physiological Integrity: Pharmacological and Parenteral Therapies 2. Which statements are true regarding the selective serotonin reuptake inhibitors (SSRIs)? (Select all that apply.) a. Avoid foods and beverages that contain tyramine. b. Monitor the patient for extrapyramidal symptoms. c. Therapeutic effects may not be seen for about 4 to 6 weeks after the medication is started. d. If the patient has been on an MAOI, a 2- to 5-week or longer time span is required before beginning an SSRI medication. e. These drugs have anticholinergic effects, including constipation, urinary retention, dry mouth, and blurred vision. f. Cogentin is often also prescribed to reduce the adverse effects that may occur. ANS: C, D Chapter 16: Psychotherapeutic Drugs 87 During SSRI medication, therapeutic effects may not be seen for 4 to 6 weeks. To prevent the potentially fatal pharmacodynamic interactions that can occur between the SSRIs and the MAOIs, a 2- to 5-week washout period is recommended between uses of these two classes of medications. The other options apply to other classes of psychotherapeutic drugs, not SSRIs. DIF: COGNITIVE LEVEL: Applying (Application) TOP: NURSING PROCESS: Planning MSC: NCLEX: Physiological Integrity: Pharmacological and Parenteral Therapies COMPLETION 1. An agitated patient is to receive an intravenous dose of diazepam (Valium). The order reads, “Give diazepam, 2 mg, IV push, now. Repeat in 15 minutes if needed.” Identify how many milliliters will the nurse administer for this dose. The medication vial contains 5 mg/mL. ANS: 0.4 mL DIF: COGNITIVE LEVEL: Applying (Application) TOP: NURSING PROCESS: Implementation MSC: NCLEX: Physiological Integrity: Pharmacological and Parenteral Therapies Chapter 17: Substance Use Disorder 90 A variety of serious neurologic and mental disorders, such as Korsakoff’s psychosis and Wernicke’s encephalopathy, as well as cirrhosis of the liver, may occur with chronic use of alcohol. Renal failure, cerebrovascular accident, and Alzheimer’s disease are not associated directly with chronic use of alcohol. DIF: COGNITIVE LEVEL: Understanding (Comprehension) TOP: NURSING PROCESS: Assessment MSC: NCLEX: Physiological Integrity: Physiological Adaptation 7. A patient is being treated for ethanol alcohol abuse in a rehabilitation center. The nurse will include which information when teaching him about disulfiram (Antabuse) therapy? a. He should not smoke cigarettes while on this drug. b. He needs to know about the common over-the-counter substances that contain alcohol. c. This drug will cause the same effects as the alcohol did, without the euphoric effects. d. Mouthwashes and cough medicines that contain alcohol are safe because they are used in small amounts. ANS: B The use of disulfiram (Antabuse) with alcohol-containing over-the-counter products will elicit severe adverse reactions. As little as 7 mL of alcohol may cause symptoms in a sensitive person. Cigarette smoking does not cause problems when taking disulfiram. Disulfiram does not have the same effects as alcohol. DIF: COGNITIVE LEVEL: Applying (Application) TOP: NURSING PROCESS: Implementation MSC: NCLEX: Physiological Integrity: Pharmacological and Parenteral Therapies 8. The nurse is conducting a smoking-cessation program. Which statement regarding drugs used in cigarette-smoking–cessation programs is true? a. Rapid chewing of the nicotine gum releases an immediate dose of nicotine. b. Quick relief from withdrawal symptoms is most easily achieved by using a transdermal patch. c. Compliance with treatment is higher with use of the gum rather than the transdermal patch. d. The nicotine gum can be used only up to six times per day. ANS: A Quick or acute relief from withdrawal symptoms is most easily achieved with the use of the gum because rapid chewing of the gum produces an immediate dose of nicotine. However, treatment compliance is higher with the use of the transdermal patch system. Nicotine gum can be used whenever the patient has a strong urge to smoke. DIF: COGNITIVE LEVEL: Applying (Application) TOP: NURSING PROCESS: Implementation MSC: NCLEX: Physiological Integrity: Pharmacological and Parenteral Therapies Chapter 17: Substance Use Disorder 91 MULTIPLE RESPONSE 1. A nurse is providing teaching for a patient who will be taking varenicline (Chantix) as part of a smoking-cessation program. Which teaching points are appropriate for a patient taking this medication? (Select all that apply.) a. This drug is available as a chewing gum that can be taken to reduce cravings. b. Use caution when driving because drowsiness may be a problem. c. There have been very few adverse effects reported for this drug. d. Notify the prescriber immediately if feelings of sadness or thoughts of suicide occur. e. Avoid caffeine while on this drug. ANS: B, D Patients taking varenicline have reported drowsiness, which has prompted the U.S. Food and Drug Administration (FDA) to recommend caution when driving and engaging in other potentially hazardous activities until the patient can determine how the drug affects his or her mental status. In addition, the FDA has warned about psychiatric symptoms including agitation, depression, and suicidality. Varenicline is an oral tablet, and common adverse effects include nausea, vomiting, headache, and insomnia. There are no cautions about taking caffeine while on this drug. DIF: COGNITIVE LEVEL: Applying (Application) TOP: NURSING PROCESS: Implementation MSC: NCLEX: Physiological Integrity: Reduction of Risk Potential 2. A patient has been taking disulfiram (Antabuse) as part of his rehabilitation therapy. However, this evening, he attended a party and drank half a beer. As a result, he became ill and his friends took him to the emergency department. The nurse will look for which adverse effects associated with acetaldehyde syndrome? (Select all that apply.) a. Euphoria b. Severe vomiting c. Diarrhea d. Pulsating headache e. Difficulty breathing f. Sweating ANS: B, D, E, F Acetaldehyde syndrome results when alcohol is taken while on disulfiram (Antabuse) therapy. Adverse effects include CNS effects (pulsating headache, sweating, marked uneasiness, weakness, vertigo, others); GI effects (nausea, copious vomiting, thirst); and difficulty breathing. Cardiovascular effects also occur; see Table 17-2. Euphoria and diarrhea are not adverse effects associated with acetaldehyde syndrome. DIF: COGNITIVE LEVEL: Applying (Application) TOP: NURSING PROCESS: Implementation MSC: NCLEX: Physiological Integrity: Physiological Adaptation Chapter 17: Substance Use Disorder 92 3. The nurse is monitoring a patient who is experiencing severe ethanol withdrawal. Which are signs and symptoms of severe ethanol withdrawal? (Select all that apply.) a. Agitation b. Drowsiness c. Tremors d. Systolic blood pressure higher than 200 mm Hg e. Temperature over 100° F (37.7° C) f. Pulse rate 110 beats/min ANS: A, C, D Signs and symptoms of severe ethanol withdrawal (delirium tremens) include systolic blood pressure higher than 200 mm Hg, diastolic blood pressure higher than 140 mm Hg, pulse rate higher than 140 beats/min, temperature above 101° F (38.3° C), tremors, insomnia, and agitation. See Box 17-6 for all signs and symptoms of ethanol withdrawal. DIF: COGNITIVE LEVEL: Applying (Application) TOP: NURSING PROCESS: Assessment MSC: NCLEX: Physiological Integrity: Physiological Adaptation Chapter 18: Adrenergic Drugs 95 7. A 14-year-old patient has been treated for asthma for almost 4 months. Two weeks ago, she was given salmeterol as part of her medication regimen. However, her mother has called the clinic to report that it does not seem to work when her daughter is having an asthma attack. Which response by the nurse is appropriate? a. “It takes time for a therapeutic response to develop.” b. “She is too young for this particular medication; it will be changed.” c. “She needs to take up to two puffs every 4 hours to ensure adequate blood levels.” d. “This medication is indicated for prevention of bronchospasms, not for relief of acute symptoms.” ANS: D Salmeterol is indicated for the prevention of bronchospasms, not treatment of acute symptoms. The dosage is usually two puffs twice daily, 12 hours apart, for maintenance effects in patients older than 12 years of age. The other options are incorrect. DIF: COGNITIVE LEVEL: Applying (Application) TOP: NURSING PROCESS: Implementation MSC: NCLEX: Physiological Integrity: Pharmacological and Parenteral Therapies 8. A hospitalized patient is experiencing a severe anaphylactic reaction to a dose of intravenous penicillin. Which drug will the nurse expect to use to treat this condition? a. Ephedra b. Epinephrine c. Phenylephrine d. Pseudoephedrine ANS: B Epinephrine is the drug of choice for the treatment of anaphylaxis. The other drugs listed are incorrect choices. DIF: COGNITIVE LEVEL: Understanding (Comprehension) TOP: NURSING PROCESS: Planning MSC: NCLEX: Physiological Integrity: Pharmacological and Parenteral Therapies 9. The nurse recognizes that adrenergic drugs cause relaxation of the bronchi and bronchodilation by stimulating which type of receptors? a. Dopaminergic b. Beta1 adrenergic c. Beta2 adrenergic d. Alpha1 adrenergic ANS: C Stimulation of beta2-adrenergic receptors results in bronchodilation. The other choices are incorrect. DIF: COGNITIVE LEVEL: Understanding (Comprehension) TOP: NURSING PROCESS: Implementation MSC: NCLEX: Physiological Integrity: Pharmacological and Parenteral Therapies Chapter 18: Adrenergic Drugs 96 10. The nurse is preparing to administer dopamine. Which is the correct technique for administering dopamine? a. Orally b. Intravenous (IV) push injection c. Intermittent IV infusions (IV piggyback) d. Continuous IV infusion with an infusion pump ANS: D Dopamine is available only as an IV injectable drug and is given by continuous infusion, using an infusion pump. The other options are incorrect. DIF: COGNITIVE LEVEL: Applying (Application) TOP: NURSING PROCESS: Implementation MSC: NCLEX: Physiological Integrity: Pharmacological and Parenteral Therapies MULTIPLE RESPONSE 1. The nurse is presenting information to a class of students about adrenergic drugs. Which are the effects of drugs that stimulate the sympathetic nervous system? (Select all that apply.) a. Dilation of bronchioles b. Constriction of bronchioles c. Decreased heart rate d. Increased heart rate e. Dilated pupils f. Constricted pupils g. Glycogenolysis ANS: A, D, E, G Stimulation of the sympathetic nervous system causes bronchodilation, increased heart rate, pupil dilation, and glycogenolysis as well as many other effects (see Table 18-1). The other responses are effects that occur as a result of the stimulation of the parasympathetic nervous system. DIF: COGNITIVE LEVEL: Applying (Application) TOP: NURSING PROCESS: Assessment MSC: NCLEX: Physiological Integrity: Pharmacological and Parenteral Therapies COMPLETION 1. The nurse is to administer epinephrine 0.3 mg subcutaneously. The ampule contains 1 mL of medication and is labeled “Epinephrine 1:1000.” Identify how many milliliters of epinephrine will the nurse give. ANS: 0.3 mL Chapter 18: Adrenergic Drugs 97 Note that 1:1000 indicates 1 gram per 1000 mL or 1000 mg per 1000 mL, which is a concentration of 1 mg/mL. DIF: COGNITIVE LEVEL: Applying (Application) TOP: NURSING PROCESS: Implementation MSC: NCLEX: Physiological Integrity: Pharmacological and Parenteral Therapies
Docsity logo



Copyright © 2024 Ladybird Srl - Via Leonardo da Vinci 16, 10126, Torino, Italy - VAT 10816460017 - All rights reserved